Sie sind auf Seite 1von 122

MATHEMATICS

1. A business man had incurred the following expenses in his trips to the Visayan
islands: P5100.00, P4600.00, and P3200.00. What was his total expense for the trip?

A. P16000 C. P17000

B. P16700 D. P17500

2. Ms. Tonelada weighed 60 kg. She lost 4 kg on her first week of exercise, gained 2 kg
on her second week, lost 6 kg on the 3rd week and remained her weight during the
fourth week. What was her weight on the 4th week?

A. 52 kg C. 68 kg

B. 58 kg D. 72 kg

3. A group of young people from four countries gathered together for an international
conference: 40 from Manila, 60 from Japan, 35 from Thailand and 45 from Singapore.
The participants will form discussion groups with equal number of members from each
country in each group. What is the greatest number of discussion groups that can be
formed?

A. 5 C. 20

B. 15 D. 25

4. During summer, a lady visits Baguio every 6 days, and his best friend every 4 days. If
they visited Baguio last April 11, what was the earliest date did both of them visit Baguio
again?

A. April 21 C. May 5

B. April 23 D. May 11

5. In a survey to determine the reaction of people having a new GSIS card, 80% of the
2,400 people voted in favour of the new card. How many of the voters did not vote for
the new card?

A. 1920 C. 800

B. 1600 D. 480

6. Lulu spends 15% of her monthly income for her house rental, 10% for electric bill and
25% for food and other miscellaneous expenses. After paying all these expenses, she
still has P6,000 left. How much does she earn every month?

A. P15,000.00 C. P9,000.00

B. P12,000.00 D. P8,000.00

7. If 500 or 25% of a graduating class are girls, how many are graduating?
A. 2,000 C. 10,000

B. 5,000 D. 20,000
8. A man sleeps 6 hours a day. What is the ratio of the time he is asleep to the time he
is awake?

A. 1:6 C. 1:3

B. 1:4 D. 1:2

9. A recipe calls for ¾ cup of sugar. How much sugar should be used if only ½ of the
quantities given in the recipe to be prepared?

3 1
A. C.
4 2

2 3
B. D.
3 8

10. If a picture frame is 27 cm long and 18 cm wide, what is the ratio of its length to its
width?

A. 3:2 C. 3:5

B. 2:3 D. 5:3

11. Five bananas weigh as much as 3 star apples. In this rate, how many star apples
will weigh as much as 45 bananas?

A. 27 C. 33

B. 30 D. 36

12. It takes 20 men to build a building for 60 days. Assuming that all men work at this
rate, how many men will be needed to build the same building in 15 days?

A. 5 C. 100

B. 80 D. 120

13. The ratio of the number of carabaos, goats, and cows in a farm is 5:1:2.

If there are 48 animals of these kinds in his backyard, how many of them are goats?

A. 2 C. 6
B. 4 D. 8

14. A defective ruler was found to be 11.5 in long. Using this ruler, Samuel was found to
be 4 ft. tall. What is Samuel’s actual height?

A. 4 ft 2 in C. 3 ft 11.5 in

B. 4 ft 4 in D. 3 ft 10 in

15. Which of the following lengths is the longest?

A. 555 cm C. .005 km

B. 5.5 m D. 5555 mm

16. One-fifth of the width and one-fourth of the length of a rectangular cardboard is cut
off. What part of the original cardboard is the area of the remaining piece?
A. 30% of the original area C.50% of the original area

B. 40% of the original area D.60% of the original area

17. If the area of a triangle is 1 sq unit and ots height is unit, what is the length of its
base?

A. 1 unit C. 3 units

B. 2 units D. 4 units

18. What is the radius of a circle whose area is 25 cm2?

A. 25 cm C. 5 cm

B. 25 cm D. 5 cm

19. Refer to the figure. Given: m 2 = 55º and m 3 = 80. What is 4?

4 1 2 5

A. 95º C. 115º
B. 105º D. 135º

20. The sum of three consecutive integers is 96. What

are the integers?

A. 31, 32, 33 C. 30, 32, 34

B. 32, 33, 34 D. 33, 34, 35

21. What is the simplest form of the expression?

A. -2x + 4y – 17 C. 2x+ 4y – 17

B. 2x + 4y + 17 D. 2x + 4y

x2 3x
22. If x = 1 and y = - 2, what is the value of the expression −4 x+ 2 ?
2 xy y

9 7
A. - C. -
2 2

15 17
B. - D. -
2 4

23. At 25% discount, Ms. Barat paid P150.75 for a bag. What was the original price of
the bag?

A. P37.69 C. P201.00

B. P150.75 D. P603.00
24. An elevator can carry a maximum load of 605 kg. How many passengers of weight
50.5 kg each can the elevator hold?

A. 12 C. 11

B. 11.9 D. 10

25. The ratio of the number of red, green and blue balls in box is 2: 5: 6.

How many green marbles are there if there are 52 marbles in all?

A. 4 C. 20
B. 8 D. 24

1
26. Mrs. Dinah Tah Tanda divided her lot among her 4 children. The first got 3 ha,
2
1 1 2
the second 3 ha, the third 3 ha and the fourth 3 ha.
3 4 5

How big is Mrs. Tanda’s lot?

5 29
A. 12 ha C. 13 ha
14 60
B. 13 ha D. 14 ha

27. A room is 30 ft long, 25 ft wide and 14 high. If 42 balloons are inside the room, how
many cubic feet of space does this allow for each balloon?

A. 25 C. 250

B. 69 D. 690

28. What is the volume of air in an atmospheric balloon with a diameter of 24 cm?

A. 144 cm³ C. 570 cm³

B. 240 cm³ D. 2304 cm³

1
29. A photograph measuring 7 cm by 5 cm is enlarged so that the longer side is 24
2
cm. What is the length ( in cm ) of the shorter side?

A. 36 C. 6

B. 16 D. 1.6

30. A man accepts position at P14,250 basic salary with an agreement that he will
receive a 2% increase every year for 3 years. What will his salary be at the end of 3
years?

A. P14,950,00 C. P15, 122.21

B. 15,105.00 D. P16,500.00

31. In a university, the ratio of female professors to the male professors is 8:5. If there
are 75 male professors, how many are female professors?

A. 120 C. 225
C. 180 D. 375/8

2
32. A farmer can plow of a hectare in 1 hr. At this rate, how many hours will 5 farmers
3
plow the same field?

1 15
A. C.
5 2

10
B. D. 10
3

33. A building 25 m tall casts a shadow 10 m long. How long is the shadow of a 5-foot
girl standing beside the building?

A. 2 ft C. 10 ft

B. 2.5 ft D. 250 ft.

34. What is the maximum number of books, each 1.4 cm thick that can be put vertically
in a shelf which is 64 cm long?

A. 44 C. 46

B. 45 D. 64

35. If the exchange rate of US dollars to pesos is $1 = Php47.90. What is thye value of
Php1 in American cents?

A. 02 C. 2

B. 479 D. 4.79

36. In a bundle of new 100 bills, the bills are consecutively numbered RTC3432260 to
RTC343228. How much is the total amount of the bills?

A. 3200 C. 3400

B. 3300 D. 3500

37. In a card game, a player got the following scores: 35, -60, -40, 80, -100, 25, -25.
What is the final score?

A. -115 C. 85

B. -85 D 115

38. A meter stick was cut into 2 at the 25 cm mark. What is the ratio of the smaller piece
to the largest piece?

A. 1:3 C. 3:4

B. 2:5 D. 4:5

39. How many cm are there in 2 cm and 550 mm?

A. 75 C. 2055

B. 255 D. 2550
40. The first five numbers in a sequence are 2, 3, 5, 8, and 12. What is the 7 th number
in the sequence?

A. 15 C. 23

B. 19 D. 25

41. A teacher wants to group her pupils into groups of 3 or 5 or 6. However, she found
out that if she do that there always be 1 pupil left. What is the least possible number of
pupils in the class?

A. 81 C. 21

B. 31 D. 11

42. What measure of central tendency can best describe the size of t-shirts commonly
used by teen-agers?

A. mean C. mode

B. median D. both A and C

43. The following are the results of the recent achievement test in Mathematics of the
four divisions.

Division Mean Standard deviation


I 34 4.5
II 34 3.5
III 23 1.0
IV 20 2.0

Which division performed best?

A. I C. III
B. II D. IV

44. An empty bow weighs 1.3 kilos. A Math book weighs 1.5 kilos. Which expression
gives the weight of the box when filled with the ƴ Math books?

A. 1.3 ƴ + 1.5 C. 1.3 + 1.5 ƴ

B. 1.5 – 1.3 D. 1.3 + 1.5ƴ

45. A man invested Php100,000. He put part of it in a bank at 5% interest. On the other
hand, he invested the remainder in bonds with a 9% yearly return. How much did he put
in the bank if his yearly income from the two investments was Php7,400?

A. Php40,000.00 C. Php60,000.00

B. Php50,000.00 D. Php70,000.00

46. How much liquid containing 6% boric acid should to be mixed with 2 quarts of a
liquid that is 15% boric acid in order to obtain a solution that is 12% boric acid?

A. 1 quart C. 3 quarts
B. 2 quarts D. 5 quarts
47. A man, 32 years old, has son 8 years of age. In how many years will the man be
twice as old as his son?

A. 16 years C. 32 years

B. 24 years D. 48 years

48. What is the probability of getting a multiple of 3 when a die is tossed?

A. 1/6 C. 1/3

1 1
B. D.
4 2

49. In how many ways can 5 basketball players be choosen from a group of 9 players?

A. 126 C. 15, 120

B. 212 D. 362,880

50. The graph shows the number of socks, belts, handkerchiefs, and neckties sold by a
store in one week.

150
Number of items

100

50

Items Sold by Store

The names of the items are missing from the graph. Socks were the item most often
sold, and fewer neckties than any other item were sold. More belts than handkerchiefs
were sold. How many belts were sold?

A. 80 C. 120
B. 90 D. 140
SCIENCE

1. Why do seasons occur on Earth?

A. The sun rotates on its axis.

B. Earth revolves on its axis.

C. Earth’s axis is tilted.

D. Earth rotates on its axis.

2. Which part of the flower develops into fruits?

A. Ovule

B. Ovary

C. Petal

D. Anther

3. Trace the energy transformation when a flashlight is on.

A. chemical light electrical

B. electrical chemical light

C. chemical electrical light

D. electrical light chemical

4. Which of the following is the unit of structure all living things?

A. Tissue

B. Organ

C. Cell

D. Muscle

5. Which is the correct sequence of the stages of mitosis?

A. Telophase, Anaphase,Metaphase, Prophase

B. Metaphase, Anaphase, Telophase, Prophase

C. Anaphase, Prophase, Metaphase, Telophase,

D. Prophase,Metaphase, Anaphase, Telophase

6. The propulsion of jets and rockets is based on Newton’s Law of ____________.

A. Acceleration

B. Interaction

C. Inertia

D. Gravitation
7. Why are shadows formed?

A. Light travels through opague objects.

B. Light is reflected

C. Light is refracted.

D. Light travels in a straight line.

8. What produces the tremendous energy of the sun?

A. Nuclear disintegration

B. Nuclear fission

C. Nuclear fusion

D. Nuclear decomposition

9. Which has a higher specific heat?

A. Sand

B. Water

C. Soil

D. Wood

10. Sound travels fastest in _____________.

A. gas

B. liquid

C. solid

D. air

11. Which explain the absence of weather on the moon?

A. Moon rotates on its axis fast.

B. Moon revolves very fast.

C. Moon’s gravity is high.

D. Moon has no atmosphere.

12. Which law states that heat flows from bodies at high temperature to one at low temperature?

A. Thermodynamics

B. Energy change

C. Transformation of energy

D. Retention of energy.
13. Which atomic particle determines the isotope of an element?

A. Positron

B. Proton

C. Electron

D. Neutron

14. Who proposed the theory of evolution that is based on natural selection?

A. Watson

B. Darwin

C. Lamarck

D. Mendel

15. What gas is released during photosynthesis?

A. Carbon dioxide

B. Oxygen

C. Nitrogen

D. Hydrogen

16. Which instrument is used to detect earthquake intensity?

A. Anenometer

B. Barograph

C. Seismograph

D. Clinometer

17. Which of the following radioactive emissions are arranged in the order of increasing
penetrating power?

A. Alpha, Gamma, Alpha

B. Alpha, Beta, Gamma

C. Gamma, Beta, Alpha

D. Beta, Gamma, Alpha

18.When light falls on soap bubbles, the band of seven colors seen is due to ____________.

A. Difraction

B. Reflection

C. Scattering

D. Interference

19. Why can mirrors form clear images?

A. Light is regularly refracted.


B. Light is irregularly reflected.

C. The angle of incidence is equal to angle of reflection.

D. The angle of reflection is equal to the angle of refraction.

20. Why do we see the sun rising in the east?

A. Earth rotates from west to east.

B. Earth revolves from east to west.

C. Earth rotates from east to west.

D. Earth’s axis is tilted.

21. A community of living organisms interacting with their non-living environment constitutes
a/an ______________.

A. population

B. ecosystem

C. species

D. biosphere

22. Choose a possible food chain.

A. Corn rat cat

B. Corn man cat

C. Rice rat man

D. Rice cat rat

23. In the energy pyramid which is at the highest point?

A. Second order producers

B. Producers

C. First order consumers

D. third order consumers

24. Which of the following pairs demonstrates symbiosis?

A. Man and lice

B. Monkey and banana

C. Chick and corn plant

D. Butterfly and bean flower

25. The zigzag road to Baguio represents which kind of simple machine?

A. Lever

B. Wheel and axle

C. Wedge
D. Inclined Plane

26. Why does mercury in a thermometer rise when in contact with hot water?

A. heat increases the number of molecules.

B. Heat makes molecules become bigger and farther apart.

C. Heat makes molecules move faster and farther apart.

D. Heat makes the molecules move slower.

27. Which blood type is a universal donor?

A. A

B. O

C. AB

D. B

28. Which nerve transmits the message received to the center of hearing in the brain?

A. Sensory

B. Optic

C. Olfactory

D. Auditory

29. Which disease affects the air sacs of the lungs and is common among cigarette smokers?

A. Asthma

B. Tuberculosis

C. Emphysema

D. Tracheal disorder
30. What type of joint is responsible for bending our arms and legs?

A. Hinge

B. Pivot

C. Ball and socket

D. Sliding

31. Which protein found in the plasma is responsible for blood clotting?

A. Globulin

B. Albumin

C. Serum

D. Fibrinogen

32. Which part of the brain is responsible for memory and intelligence?

A. Cerebellum
B. Cerebrum

C. Medulla

D. Thalamus

33. The Philippines belong to a type of biome called ______________.

A. deciduous forest

B. taiga

C. tropical rainforest

D. grassland

34. The factors that significantly contribute to an increase in human population are__________.

A. immigration and natality

B. emigration and natality

C. immigration and mortality

D. emigration and mortality

35. What is the common cause of the temporary wilting of plants at noon?

A. Increased condensation

B. Rapid respiration

C. Rapid transferation

D. Increased oxidation

36. How can we identify a dicot leaf from a monocot leaf?

A. A dicot leaf has a smooth surface.

B. A dicot leaf has a netted venation.

37. When the stems of vines grow around poles of fences, what kind of response is exhibited?

A. Geotropism

B. Thigmotropism

C. Hydrotopism

D. Phototropism

38. Which of the following is the greatest contributor to our “greenhouse effect?”

A. Increase of oxygen

B. Leaching of the soil

C. Increase of herbivores

D. Cutting forest trees

39. Why does a tree slowly die when its bark all around it is removed?
A. The cambium is injured

B. The phloem vessels are cut.

C. The tree cannot manufacture food.

D. Its protective layer is cut.

40. Which is the method of growing vegetables and low plants in a nutrient packed, aerated
solution with no soil at all?

A. Vegetative cultivation

B. Natural farming

C. Hydroponics

D. Aerial cultivation

41. When you carry a heavy load with one arm, you tend to hold your free hand away from your
body in order to ______________.

A. change the weight of your body and load

B. change the mass of your body

C. be ready to grab something in case you fall

D. change the center of gravity of your body and load

42. Why are door knobs placed at the edge of a door rather tha in a center?

A. To increase the lever arm

B. To increase the force on the door

C. To help give more momentum to the door

D. Closer to the edge of the doorway

43. Why is the dam built wider at the bottom than at the top?

A. To withstand the greater pressure at the bottom

B. To balance the pressure at the top

C. It holds up the dam better than at the center

D. It anchors the dam better

44. Archimedes’ principle states that an object is buoyed up by a force that is equal to the
_____________.

A. volume of the fluid

B. weight of the fluid displaced

C. mass of the fluid displaced

D. mass of the object

45. When an ice cube in a glass of water melts, the water level _______________.

A. falls
B. remains the same

C. rises

D. varies

46. Two pots of the same size are filled with boiling water. One pot is colored white and the
other is black. In which pot will water cool faster?

A. Will cool at the same rate

B. In white pot

C. In black pot

D. In both pots

47. You inherited tallness from your father. Physical traits are transmitted from parents to off
springs through _______________.

A. gametes

B. chromosomes

C. alleles

D. genes

48. The handle of the spoon dipped in a cup of hot water becomes warm. What is the method of
heat transfer?

A. Condensation

B. Radiation

C. Convention

D. Conduction

49. The force required to maintain an object at a constant speed in free space is equal to
___________.

A. weight of the object

B. mass of the object

C. zero

D. force to required to stop it

50. Two people pull on a rope in a tug-of-war, each pull with a 400 N force.

What is the tension in the rope?

A. 800 N
B. 0
C. 400 N
D. 200 N

51. Why does a pressure cooker cook food faster?

I. Water boils faster


II. It raises the boiling point of water
III. It raises the cooking temperature of food.

IV. It is tightly closed

A. II and II
B. I, II and IV
C. I,II and III
D. II and IV

52. You noticed that big industries have tower-like tall stacks. What is the structure for?

A. To control emission of hazardous gasses

B. To induce rain and air pressure

C. To get enough air from the atmosphere

D. to make the interior cool

53. Steam burns are more damaging than burns caused by boiling water because stem ______.

A. has a higher temperature than boiling water

B. has more energy per kilogram than the boiling water

C. is vapor of water of water molecules during boiling

D. occupies more spaces that water during boiling

54. The removal of broken pieces of rocks from the parent rock by the action of the wind or
water is termed _____________.

A. erosion

B. deposition

C. weathering

D. silting

55. The voltage across a transformer primary coil that has 100 turns is 10 V. What is the voltage
across the secondary coil which has 20 V turns?

A. 100 V

B. 10 V

C. 2 V

D. 20 V

56. Trace the conversation of energy that occurs when plants manufacture food.

A. Radiant energy to chemical energy

B. Radiant energy to kinetic energy

C. mechanical to potential energy

D. light energy to kinetic energy

57. Riders in a bus are pushed forward during a sudden stop. Which law of motion provides an
explanation?
A. Law of Inertia

B. Law of Interaction

C. Law of Universal Gravitation

D. Hooke’s Law

58. What type of eclipse occurs when the moon is directly between the sun and the earth?

A. Partial Lunar

B. Lunar

C. Stellar

D. Solar

59. In our daily weather forecast, the region where two tropical wind systems converge,
characterized by showers and widespread thunderstorm is called ___________.

A. warm and cold fronts

B. intertropical convergent zone

C. trade winds

D. local disturbance zone

60. While traveling at night, you lost direction. Which star should you look for and in what
constellation?

I. Polaris

II. Rigel

III. Ursa Minor

IV. Ursa Major

A. I and V
B. I and II
C. II and III
D. II and IV

61. At present, a patient fights cancer either by surgery or by drug treatment called ________.

A. irradiation

B. viral treatment

C. chemotherapy

D. immune therapy

62. The U.S. was able to produce a test-tube baby through a process called ___________.

A. gene pairing

B. internal fertilization

C. protoplast fusion
D. in vitro fertilization

63. Patients suffering from malfunction of the kidney are now assisted by a machine that serves
as artificial kidney called ____________.

A. hemoscope

B. pacemaker

C. hemodializer

D. hemeostatic machine

64. The sun appears more reddish at sunset than at noon. Which of the following phenomena is
responsible for this effect?

A. Dispersion

B. Reflection

C. Scattering

D. Interference

65. Why do we hear thunder some seconds after seeing lightning?

A. Light travels fastest than sound.

B. Sound travels at 1,331 m/s.

C. Sound is released later actually.

D. light appears brighter in the sky.


SOCIAL SCIENCE

1. What does a professional code of ethics prescribe for all?


A. Moral standards and ethical behaviour
B. Rules and regulations in practicing the profession
C. Strict implementation of laws of the land
D. Civic-mindedness and social consciousness
2. “Man is to be hailed as a social being”. What does this line mean?
A. He necessarily attends social events.
B. He must introduce himself in social gatherings.
C. He does not live only for himself.
D. He does not need other peole

3. At base of a triangle that shapes us into what we are, is one of the following. Which one?

A. Environment

B. Heredity

C. Training

D. School Attended

4. What does “presumption of innocence” mean in so far as human rights is concerned?

A. A suspect is considered guilty until proven otherwise.

B. A suspect has a right to remain silent.

C. A suspect has a right to legal counsel.

D. A suspect remains innocent until proven guilty

5. No person may be elected as President of the Philippines unless he/she is a __________.

A. natural born citizen

B. resident for at least 2 year

C. at least 21 years old

D. professional

6. What do we call a typical social structure which describes a pattern through which
relationship at work are ordered?

A. Technological

B. Communistic

C. Capitalistic

D. Bureaucratic

7. A society cannot exist without a government. Which word is synonymous with government?

A. Domination

B. Authority

C. Restraint
D. Check-and-Balance

8. What does a “nuclear family” consists of?

A. Grandfathers and grandmothers

B. Father, mother, sons and daughters

C. Sons, daughters and cousins

D. Son’s, daughters and in-laws

9. What does “capital” in economics refer to?

A. Investment and loss computed

B. Outcome of a business transaction

C. Money/machines invested to transact business

10. Which of the four pillars of education for the 21st century is crucial in the light of conflicts
between Israel and Palestine?

A. Learning to do

B. Learning to be

C. Learning to live together

D. Learning to know

11. Which is the most important element of a state?

A. People

B. Government

C. Territory

D. Sovereignty

12. According to the Universal Declaration of Human Rights , what are the three fundamental
human rights?

A. Life, liberty and self-development

B. Life, liberty and security

C. Life, dignity and security

D. Life, dignity and sefl-development

13. What form of government is characterized by the separation of power?

A. Parliamentary

B. Presidential

C. Aristocracy

D. Monarchical

14. Who does the canvassing of votes for President and Vice President in every election?
A. Comelec and Chief Justice

B. Senate and Congress

C. Supreme Court and Congress

D. Joint committee of Congress

15. All appropriations, revenue or tariff bills shall originate exclusively from the _____________.

A. Senate

B. Sangguniang Panlalawigan

C. House of Representatives

D. Department of Budget

16. What occurs when the government spends more than what it collects in the form of taxes?

A. Excessive allotment

B. Fiscal overspending

C. Fiscal deficit

D. Erroneous appropriations

17. Which department has the authority to make laws and to alter them when needed?

A. Executive

B. Legislative

C. Judicial

D. Administrative

18. When can we say that a person has values?

A. Lives a life of integrity, sincerity and commitment

B. Enjoys life of leisure and pleasure

C. Works towards self-aggrandizement

D. Lives a life of joy in the company of others

19. What is the type of cooperative which promotes thrift among members and creates funds in
order to ba able to grant loans?

A. Service

B. Producers

C. Consumers

D. Credits

20. One of the major functions of taxes is to finance real government expenditures. This function
can simply be stated as __________________.

A. promote development
B. to secure price stability

C. to raise revenues

D. raise national security

21. Who is the general who said “I shall return!”? A monument In his honor stands in Corregidor
by the bay.

A. William H. Taft

B. Douglas McArthur

C. John Dewey

D. Schurmann

22. A bill becomes a law even if not signed by the President.

A. 60

B. 40

C. 20

D. 30

23. Which refers to the act of the President to stay the execution of a convict?

A. Commutation

B. Amnesty

C. Pardon

D. Reprieve

24. Who has the power to declare the existence of a state of war?

A. Chief Justise

B. President

C. Senate President
D. Congress

25. A losing senatorial candidate may file a protest against the winner. What body will assume
jurisdiction over the case?

A. Supreme Court

B. Electoral Tribunal

C. Comelec

D. Regional Trial Court

26. Which has the main vision of the DECS when it formulated Values Education Program?

A. A productive society

B. A globally competitive nation

C. a just and humane society


D. An industrialized nation

27. In 1565 Legaspi concluded a blood compact with the chief of Bohol. Who is reffered to?

A. Raja Sulayman

B. Sikatuna

C. Lakandula

D. Rajah Tupas

28. How is the crime of rape classified?

A. Heinous

B. Homicide

C. Slander

D. Malicious Mischief

29. When is the United Nations Day celebrated?

A. March 17

B. October 21

C. September 21

D. October 24

30. Which landmark located in New York City was attached by terrorists on September 11,
2001?

A. Sears Tower

B. Empire State Building

C. Rockefeler Center

D. World Trade Center

31. The Japanese successful invasion was climaxed by the surrender of the joint Filipino-
American forces on May 6, 1942. Where did it happen?

A. Bataan

B. Corregidor

C. Capas

D. Manila

32. the Philippines objected to the advisory on travels to Mindanao by the Austrilian
government. What should the government do?

A. Declare war against the country

B. File diplomatic protest

C. Recall all nationals

D. Arrests their nationals


33. Foreign investors are discouraged in doing business with our country . What condition is
being blamed for this?

A. Deteriorating peace and order

B. Poor technology

C. High tariffs

D. Poor standards

34. Which terms refers to the measures undertaken by the government which would remove
anything that hinders the entry of foreign products, services and capital between countries?

A. Liberalization

B. Globalization

C. Privatization

D. Internalization

35. Which is one of the world Heritage Sites of our country?

A. Intramurous Walled City

B. Palawan Underground River

C. Banawe Rice Terraces

D. Batanes Stone Houses

36. What do we call the sum of the money collected for our use of a road, bridge and highway?

A. Tax

B. Assessment

C. Penalty

D. Toll

37. Which term refers to duties payable on goods, whether imported or exported?

A. Assessment

B. Tariff

C. Subsidiary

D. Revenue

38. What is the principal means through which culture is transmitted from generation to
generation?

A. Community action

B. Education

C. Family

D. Language
39. When people are experiencing problems which may be considered global in perspective,
what set of values must they process in order to cope successfully?

A. Personal authority, independence, vision

B. Security, survival, self-restraint

C. Intimacy, independence, security

D. Work ethics, positive vision and self-acceptance

40. Material culture refers to the tangible and concrete objects produced by man.

Which is a material culture?

A. Stone walls of Tasadays


B. Ceremonies of Ivatans
C. Sinulog celebrations
D. Mariones festival

41. Which human need is considered basic?

A. Pleasure

B. Security

C. Wealth

D. Order

42. Which factors determine population growth?

A. Industry, investment and societal demands

B. Birth rate, death rate and migration trends

C. Mortality, birth control and economic growth

D. Laws, government and civic responsibility

43. Which term refers to the social aspects of sex or to socially defined roles and expectations
that are associated with sex?

A. Social differentiation

B. Social class

C. Gender

D. Cultural context

44. To which type of political system do we belong?

A. Colonialism

B. Totalitarianism

C. Democracy

D. Capitalism

45. In which form of government does power or authority resides in a few persons who govern
for their own interest?
A. Oligarchy

B. Monarchy

C. Democracy

D. Parliamentary

46. Which is the biggest and most populated island of the Philippine Archipelago?

A. Mindanao

B. Luzon

C. Cebu

D. Leyte

47. In what part of the globe can we trace the International Date Line?

A. North Pole

B. Atlantic Ocean

C. Pacific Ocean

D. South Pole

48. Which of the following sountries is not part of ASEAN (Southest Asian Nations)?

A. Singapore

B. Thailand

C. Malaysia

D. Taiwan

49. What is the present population of the Philippines ?

A. 83 m

B. 73 m

C. 60 m

D. 90 m

50. Which cities are three leading financial centers of the world?

A. Hongkong, Paris, Rome

B. London, New York, Tokyo

C. Beijing, Sydney, Seoul

D. Singapore, Berlin, Moscow

51. Among the developing Asian nations, which of the following is widely promoted to be the
better solutions to poverty and unemployment?

A. Privatization
B. Urbanization

C. Industrialization

D. Importation

52. Which is the oldest University in the Philippines which founded by the Dominicans in 1611?

A. Ateneo de Manila

B. University of the Philippines

C. University of Santo Thomas

D. San Juan de Letran

53. Who is the President who is known for his “Filipino First Policy” and Austerity Program?

A. Diosdado Macapagal

B. Carlos P. Garcia

C. Manuel Roxas

D. Ramon Magsaysay

54. Which city suffered from the first atomic bomb on August 6, 1945?

A. Hiroshima

B. Chernobyl

C. Moscow

D. Nagasaki

55. Pre-colonial trade in the Philippines was prosperous. What system did they use for their
business transactions?

A. Money

B. Barter

C. Credit

D. Banking

56. Which tools were made of during the Lower Paleolithic period?

A. Stones

B. Bronze

C. Iron

D. Ceramics

ENGLISH

1. He ran for the bus and ________ on.

A. had jump C. jumps


B. jump D. jumped

2. The test was hard for Paul and ____________.

A. me C. myself

B. I D. himself

3. The flower smells _____________.

A. sweetly C. sweeter

B. sweet D. more sweet

4. The suggestions of the employees __________ appropriate.

A. seems C. is

B. seem D. is very

5. He is the _____________ of the two brothers.

A. most tall C. very tall

B. tallest D. taller

6. After __________ all days, she finally saw the lake.

A. walking C. having walked

B. had walked D. have walked

7. Be sure to fill ___________ the forms correctly.

A. over C. with

B. on D. in

8. We had the ___________ news when he came.

A. latest C. very last

B. last D. last of all

9. I ___________ gone with you.

A. have C. had

B. should have D. am

10. I ___________ walked two kilometres by the time you catch up with me.

A. have C. had

B. should D. had

11. If I ___________ President, I would live in Malacanang

A. was C. should be

B. am D. were

12. The means of transportation ________ dramatically since the end of the 20th century.
A. have changed C. will change

B. has changed D. will have changed

13. If I ____________ same problems you had as a child, I might not have succeeded in life as
well as you have.

A. should C. have

B. had D. would have

14. If I could speak English well, I ___________ next year studying in the States.

A. has spend C. would spend

B. would have spent D. will spent

15. I live in a house ___________ of the road.

A. in the side C. at the side

B. beside D. by the side

16. The boy likes to slide ____________ the banister.

A. beside C. along

B. by D. down

17. It is cooler ____________ part of the garden.

A. in the shaded C. render the shade

B. in the shady D. on the shady

18. I cannot sleep ____________ I am surrounded with books.

A. if C. when

B. unless D. except

19. The hawk glided _____________ the mountain.

A. in C. toward

B. into D. by

20. We should work for the total ban ____________ of pesticides.

A. with C. on the use

B. on using D. by the use

21. The Rizal Day celebration reminds us about heroes worth ______________.

A. appreciating C. reading about

B. emulating D. studying

22. Mary eyes blurred because she _________ at the computer for five straight hours.

A. was C. have been sitting


B. had been sitting D. would have been

23. A good writer supports his generalizations with __________ examples.

A. unfamiliar C. abstract

B. convenient D. concrete

24. Students ___________ near the registrar’s office to ask for their schedule.

A. crowd C. stay idling

B. congregate D. disperse

25. She will retire soon because she __________ working hard so long.

A. has been C. have been

B. had been D. was

26. “He is the black sheep in the family.” Which figure of speech is used?

A. Synonym C. Metaphor

B. Simile D. Hyperbole

27. The hostages are free. They paid their way. What does this statement tell us?

A. They escaped after paying.

B. After paying ransom they were released

C. The negotiated peacefully.

D. They were freed without ransom.

28. October 10 is a “red letter day” for the Chinese. Classify this expression.

A. Idiom C. Metaphor

B. Simile D. Synonyms

29. Space age ________ on October 21, 1957 when Sputnik was launched by the Soviet Union.

A. commenced C. ended

B. cited D. flourished

30. When he completed the assignments, he _________ it was well done.

A. realize C. was realizing

B. had realize D. realized

31. The slow traffic __________ me.

A. aggravates C. worsens

B. irritates D. maddens

32. He was as helpless as a child. Classify this statement.

A. Simile C. Synonym
B. Idioms D. Metaphor

33. The statement, “ She is a rose” is an example of a _______________.

A. idiom C. simile

B. metaphor D. hyperbole

34. Because the moon rotates on its axis at the same as it ______________ around the earth,
we see the same side always.

A. revolve C. has been revolving

B. is revolving D. revolves

35. Children who have been exposed too much to TV suffer from unusually short ___________.

A. learning C. concentration

B. understanding D. attention span

36. People who are too _______ liable to be deceived by unscrupulous individuals.

A. demanding C. cynical

B. credulous D. wary

37. News travels as fast as the wind, This statement is a/an ________________.

A. simile C. idiom

B. metaphor D. synonym

38. She wrote a “first rate” report. What does it say about the report?

A. Ordinary C. Mediocre

B. Metaphor D. Synonym

39. “A man is known by the company he keeps”. This is an example of a ____________.

A. report C. poem

B. rule D. proverb

40. His apocalyptic views seemed to doom the human race. This is to be taken as __________.

A. apathetic C. prophetic

B. lethargic D. spiritless

41. Our guest of honor was a loquacious speaker. This means _______________.

A. reserved C. reticent

B. verbose D. spiritless

42. Except for the profanity, I liked the story. What is the story about?

A. Shamelessness C. Worldliness

B. Kinds words D. Obscenities

43. She has a penchant for red accessories. This indicates____________.


A. strong aversion C. fondness

B. dislike D. avoidance

44. Stop shilly-shallying and make up your mind. This serves as warning not to _____________.

A. go-ahead C. decide

B. hesitate D. continue

45. My grandma had some lucid moments before she died. This means ____________.

A. confused C. clear thinking

B. ambiguous D. sad

46. The speech was full of gobbledygook. What does it mean?

A. Jargon C. Ideas

B. Sense Facts

47. “ She is a vision of feminine pulchritude.” This stands for ____________.

A. homeliness C. plain

B. loveliness D. simple

48. The statue was erected in __________ in to war heroes.

A. protection C. irreverence

B. defense D. homage

49. She was a “Phantom of Delight” connotes that the woman is ___________.

A. real C. an illusion

B. lovely D. virtuo

50. The line, “Under the bludgeoning of chance. My head is bloody but unbowed” depicts the
person’s _____________.

A. determination C. confidence

B. courage D. optisim

51. Who is the author of “ Whitout Seeing the Drawn”?

A. Maximo Ramos C. Paz M. Benitez

B. Magdalena Jalandoni D. Stevan Javellana

52. Which among these is Nick Joaqin’s pen name?

A. Douglas Quijano C. Mang Niko

B. Quijano de Manila D. Santa Claus

53. “It is only with the heart that one can see rightly.” This line means ____________.

A. important things are invisible to the heart

B. only the heart can see the beautiful things in life


C. the heart is always correct about everything

D. the heart is visible if one is kind

54. “I prefer a government run like hell by Filipinos rather than one run by foreigners.” Who said
this famous statement?

A. Manuel L. Quezon C. Apolinario Mabini

B. Jose Rizal D. Emilio Aguinaldo

55. Our favourite song goes,

Each man is an island, No man stands alone,

Each man’s joy is joy to me

Each man’s grief is my own.

What does this song express?

A. Solitude C. Brotherhood

B. Independence D. Individualization

56. “My loyalty to my party ends where my loyalty to my country begins.”

Who said this famous quotation?

A. Manuel L. Quezon C. Carlos P. Romulo

B. Manuel Roxas D. Ferdinand E. Marcos

57. What does the expression “love begets love” mean?

A. Love is offered, not waiting for love in return.

B. If you offer love, you will be loved in return.

C. Don’t expect love, though you love others.

D. Love is not freely expected from everyone.

58. Who said “The Filipinos is worth dying for?”

A. Benigno Aquino Sr. C. Andres Bonifacio

B. Jose Rizal D. Carlos P. Garcia

59. The famous line goes, “Anything of beauty is a ________ forever.”

A. joy C. vision

B. dream D. good

LITERATURE

60. Archimedes shouted “Eureka!” What did we take this to mean?


A. Wonderful C. Mystery

B. Enjoyment D. Discovery

61. “I die when just the drawn breaks to herald the day.” This is a famous line quote from

A. Andres Bonifacio C. Benigno Aquino

B. Jose P. Rizal D. Manuel L. Quezon

62. Shakespeare once said, “My honor is my life, both grow as one, take honor from me and my
life is gone.” What is the correct interpretation for this?

A. His life is determined by his honor.

B. He values honor as much as his life.

C. He prefers honor instead life.

D. He values life before honor.

63. “Run from anything that gives you the evil thought…. But say close to anything that makes
you want to do right.”

It is about defeating __________.

A. wrong decisions C. temptation

B. righteousness D. faith

64. Thomas Cycle said “The man without a purpose is like a ship without rudder-a waif….” It
could be interpreted as ___________.

A. there should be sufficient energy in one’s life.

B. one’s life doesn’t need to be controlled

C. direction can never come into one’s life

D. there should be a strong driving force in one’s life.

65. The bible tell us, “There is a right time and a right way to do everything.”

A. Plan well your words and deeds.

B. You can determine your own time

C. Your time is yours and nobody else.

D. Say and do things anytime.

66. Our commitments can develop us or they can destroy us, but either way, they will define us.
Choose the line below that has the same meaning.

A. Our lives are shaped by what we are committed to.

B. Our commitments need not to be planned way ahead.

C. We must make full commitment to worldly goals

D. Half-hearted commitments is what we need today.

67. Who wrote about the ill fated love affair of Jose Rizal and Leonor Rivera?
A. Francisco B. Icasiano C. Arturo R. Rator

B. Nick Joaquin D. Zoilo Galang

68. What is known as a body of poetry, which originated from among the common people of
England?

A. Ode C. Lyric poems

B. Ballad D. Elegy

69. What is a distinguishing characteristic of a sonnet?

A. 31-syllable poem C. 16-sline lyric

B. 14-line lyric D. 17-syllable poem

70. “If eyes were made for seeing then ____________ is its own excuse for being.”

A. beauty C. friendship

B. love D. joy

71. What season is it, with the line, “He ruffles every lily-pond? Where blossoms kiss and part.”

A. Spring C. Autumn

B .Winter D. Summer

72. The line from the Koran, “Woe to every backbiter,” talks of ____________.

A. anguish for those who defend unrighteousness

B. misery for those who talks ill of others

C. punishment for those who say good things

D. sorrow for those who oppose other’s opinion

73. “The Lord is my Shepherd; I shall not want….” is an expression of one’s

A. love for self C. blind ignorance

B. poverty D. total submission to God

74. Who is considered the most outstanding among Filipino Playwrights in English?

A. Wilfredo Ma. Guerrero C. Severino Montano

B. Carlos P. Romulo D. Leon Ma. Guerrero

PILIPINO

1.Tukuyin ang sugnayan na makapag-iisa?


Kung magkakasundo tayo, ikaw ang mamumuno at ako naman ang tagasunod.

A. Kung magkakasundo tayo C. ang magiging tagasunod

B. ako naman D. ikaw ang mamumuno

2. Ano ang salin ng “Bring home the bacon”?

A. Mag-uwi ng bacon C. Bumili ng bacon

B. Mag-uwi ng panalo D. Dalhin ang bacon

3. Ano ang uri ng pangungusap na ito?

UMULAN NA.

A. Pormulasyong panlipunan C. Penomenal

B. Panawag D. Sambitla

4. Ano ang kayarian ng pangungusap na ito?

Nakipagkita sa Pangulo ang mga senador at kinatawan ng iba’t ibang samahan.

A. Tambalan C. Hugnayan

B. Langkapan D. Payak

5. Ibigay ang paksa ng pangungusap.

Nabasa ko sa isang aklat ang kasaysayan ng ating bansa.

A. Kasaysayan C. Bansa
B. Aklat D. Nabasa

6. Ano ang kahulugan ng sumusunod?

ILISTA MO NA LAMANG SA TUBIG ANG AKING UTANG.

A. Kalimutan na ang utang. C. Tubig ang listahan

B. Magbabayad din ng utang D. Nasa tubig ang utang

7. Alin dito ang mga salita na ang kahulugan ay iba sa kahulugan ng mga salitang binubuo?

A. Tayutay C. Salitang upemisitiko

B. Ekspresyong idiomatiko D. Pahayag

8. Ibigay ang kayarian ng paksa ng sumusunod na pangungusap.

“Ang para sayo ay kunin mo na.”

A. Pang-ukol C. Pantukoy

B. Pangatnig D. Pang-uri

9. Ano ang pokus ng pandiwa sa pangungusap na ito?

Pinasyalan ng magkakaibigan ang Splash Mountain noong isang buwan

A. Direksyon C. Tagapangtanggap
B. Tagaganap D. Sanhi
10. “ Kung ano ang bukangbibig siyang laman ng dibdib.” Ito ay isang uri ng _____________.

A. Tula C. tugmaan

C. salawikain D. bugtong

11. “Iyon lamang nakakaranas ng mga lihim na kalungkutan ang maaaring maakilala ng mga
lihim na kaligayahan.”

A. Kapangitan ng buhay C. Paghihikahos sa buhay

B. Kalungkutan ang buhay D. Kagandahan ng buhay

12. Alin sa mga sumusunod angpngungusap na may paska?

A. May pasok ba bukas?

B. Kay ganda ng paglubog ng araw.

C. Nagbabasa sila sa aklatan.

D. Mainit ngayon.

13. Sa taas ng mga bilihin ngayon kahit KAHIT KAHIG KA NANG KAHIG ay wala pa ring
maipon. Ano ang ibig sabihin?

A. Hanap ng hanap C. Gastos nang gastos

B. Tago ng tago D. Trabo ng trabaho

14. Ang pahayagan ay hindi dapat maglathala ng anumang uri ng pagbibintang na makakasira
sa reputasyon nang di muna nagbibigay ng pagkakataon sa nasasakdal na marinig ang
kanyang panig. Ito ay ____________.

A. walang kinikilingan C. makatarungan pakikitungo

B. kalayaan ng pagpapahayag D. magandang kaasalan

15. “You can count on me,” Ang pinakamalapit na salin nito ay ______________.

A. bilangin mo kami C. ibilang mo ako

B. bilangin mo ako D. maaasahan mo ako

16. Anong uri ng tayutay ang pahayag na ito? “Ang kabutihan mo sa buhay ang magiging
hakbang sa pag-unlad

A. Pagwawangis C. Pagtutulad

B. Pagpapalit-tawag D. Personipikasyon

17. “Malalim ang bulsa,” ng kanyang Nanay. Ang ibig sabihin nito ay __________.

A. walang pera C. mapagbigay

B. mapera D. kuripot

18. Siya ay May Kutsarang Pilak ng ipinanganak. Ito ay nangangahulugang _____________.

A. tahimik C. mayaman

B. masalita D. mahirap
19. “Ang hindi magmahal sa sariling wika, mahigit sa hayop at malansang isda.” Ito ay hango sa
tula ni ______________.

A. Apolinario Mabini C. Andres Bonifacio

B. Jose Rizal D. Manuel Quezon

20. Tukuyin ang uri ng pangungusap na itong walang paksa. “Walang anuman.”

A. Panagot C. Eksistensyal

B. Patanong D. Pormulasyong panlipunan

21. Ano ang tawag sa awit ng pag-ibig

A. Ihiman C. Uyayi

B. Kundiman D. Tagumpay

22. Alin pinakatamang pahayag?

A. Sa Luneta namasyal ang mag-anak

B. Namasyal ang mag-anak sa Luneta

C. Ang mag-anak ay namasyal sa Luneta

D. Nag-Luneta ang mag-anak

23. ang paggamit ng PUTTING KALAPATI bilang sagisag ng kapayapaan ay tinatawag na


____________.

A. imahinismo C. naturalism

B. simbolismo D. realism

24. “Tinaga ko sa gubat

Sa bayan nag-iiyak.”

Ito ay halimbawa ng isang _________________.

A. kawikaan C. bugtong

B. kasabihan D. salawikain

25. “Ang panalangin ay mabisang sandata sa buhay.”

A. onomatopiya C. pagwawangis

B. pagtutulad D. personipikasyon

26. Alin ang pinakamalpit na salin ng “You are the apple of my eye.”

A. Ikaw ay kasing ganda ng mansanas.

B. Ikaw ay maganda sa panangin

C. Ibibili kita ng mansanas.

D. Ikaw ang paborito.

27. Ang salitang nagtataglay ng KAMBAL-KATINIG ay ________________.


A. mag-aaral C. silid

B. klase D. guro

28. Damhin mo ang init ng pagmamahal ng iyong magulang. Ang salitang nasa malaking titik
ang nasa pandiwang di karaniwan na ____________.

A. may pungos C. may kaltas

B. may katulad D. may palit

29. Ibigay ang pokus ng pandiwa ng sumusunod na pangungusap. “Bumili ang Ate ng ulam.”

A. Pokus sa actor C. Pokus sa layon

B. Pokus sa sanhi D. Pokus sa benepaktibo

30. Ibigay ang kayarian ng paksa ng sumusunod na pangungusap. “Ang para sa iyo ay kunin
mo na.”

A. Pang-ukol C. Pantukoy

B. Pangatnig D. Pang-uri

31. Pare-pareho kayo. Wala akong mapagpilian,” wika ng isang gurong puno ng galit sa mga
lalaking mag-aarl.

A. Walang gana sa pagtuturo.

B. Nawalan ng tiwala sa mga lalaki

C. Walang pagtingin sa nga lalaki.

D. Hindi pare-pareho ang mga lalaki

32. Ang pagtama sa lotto ay parang isang “suntok sa buwan.” Ano ang kahulugan nito?

A. Maaari C. Imposible

B. Pangarap D. Maaabot

33. Ito ay kuro-kuro ng patnugot hinggil sa mahalaga at napapanahong isyu sa araw na iyon.
Ano ito?

A. Pilak C. Balita

B. Lathalain D. Editoryal

34. “Panahon na upang magdilat ng mata at makisangkot sa mga usapin.” Ito’y nagpapahiwatig
na _______________.

A. kalimutan ang usapin

B. magising sa katotohanan

C. idilat ang mga mata

D. umiwas sa usapin

35. Alin sa mga sumusunod ang hindi maituturing na pangungusap?

A. Mayroon bang dadalo? C. Maraming salamat.


B. Umaaraw ngayon D. Kung aalis ka.

36. “Ang lumalakad ng mabilis, kung matinik ay malalim.”

Ano ang ibig sabihin ng paalaalang ito?

A. Mag-isip na mabuti sa paglalakad.


B. Mag-isip ng malalim kung naglalakad.
C. Maaaksidente ang mabilis lumakad.
D. Huwag mag-isip kung naglalakad.

37. Mahusay “maglubid ng buhangin” ang taong gipit. Ano ang ibig sabihin?

A. Magyabang C. Magpaikot-ikot

B. Magsinungaling D. Magtali

38. Huwag “pagbuhutan ng kamay” ang batang walang kalaban-laban. Ano ang ibig sabihin?

A. Itali ang kamay C. Pagbuhatin ng mabigat

B. Pagtrabahuin D. Saktan

39. “Mayroon bang pag-asa na umunlad pa?” ano ang uri ng pangungusap na ito na walang
paksa?

A. Eksistensyal C. Pamanahon

B. Pormulasyong panlipunan D. Sambitla

40. “Inaawitaan anon g mga alo sa dalampasigan.” Ito ay halimbawa ng____________.

A. paglilipat-wika C. pagtutulad

B. pagwawangis D. personipikasyon

41. Sino ang nagsabi ng sumusunod, “Ang hindi magmahal sa sariling wika ay higit pa anh
amoy sa malansang isda?”

A. Jose Rizal C. Apolinario Mabini

B. Gregorio del Pilar D. Marcelo H. Del Pilar

42. “Sa isang saglit ay naihahambing ko ang aking sarili sa isang paru-parong palipat-lipat sa
mga bulaklak.” Anong uri ng pananalita ito?

A. Patulad C. Padiwantao

B. Pawangis D. Metonomiya

43. Iyon lamang nakakaranas ng mga lihim na kalungkutan ang maaaring makakilala ng lihim
na kaligayahan. Ang mga pahayag na ito ay nagsasaad ng ______________.

A. paghihikahos sa buhay C. kapangitan ng buhay

B. kalungkutan ng buhay D. kagandahan ng buhay

44. Siya ay kauna-unahang itinanghal na hari ng balagtasan. Sino siya?

A. Ildefonso Santos C. Florentino Collantes

B. Lope K. Santos D. Jose Corazon de Jesus


45. Siya ay kauna-unahang nagsalin sa Tagalog ng Mi Ultimo Adios ni Jose Rizal:

A. Jose Gatmaitan C. Apolinario Mabini

B. Andres Bonifacio D. Jose Corazon de Jesus

46. Ito ay isang tulang pasalaysay na kinapapalooban ng pakikipagsapalaran, pamumuhay at


kabayanihan ng isang tauhang may pambihirang katangian at kasamang kababalaghan. Piliin
sa mga sumusunod.

A. Epiko C. Bugtong

B. Alamat D. Awit

47.Saan inaawit ang awiting-bayan na “Atin Cu Pong Singsing”?

A. Bikol C. Pampanga

B. Ilocos D. Leyte

48. Mahusay magtago ng lihim ang mga taong “mabigat ang bibig.” Ano ang ibig sabihin nito?

A. Hindi madaldal C. Malaki ang bibig

B. Mahiyain D. Tahimik

49. Sino ang unang bumuo ng titik ng ating pambansang awit na may pamagat ng Himno
Nacional Filipino?

A. Graciano Lopez Jaena C. Andres Bonifacio

B. Jose Palma D. Marcelo H. Del Pilar

50. Alin ang pahayang itinatag ni Marcelo H. Del Pilar upang ilathala ang maling pagbatikos sa
maling pamamahala ng mga Kastila?

A. El Porvenir C. La Solidaridad

B. Diariong Tagalog D. EL Resumen

51. “Kung sa langit ay nabubuhay ang sa lupa ay namamatay ano’t kinatatakutan ang oras ng
kamatayan.” Anong damdamin ang ipinapahayag nito?

A. Maging matapang C. Mapagbigay

B. Maging mapagpasensiya D. Matatag sa buhay

52. Kailan ginagamit ang isang pananalitang patulad?

A. Sa paghahambing ng dalawang kaisipan

B. Sa paghahambing ng dalawang magkaibigang bagay

C. Sa paghahambing ng magkatulad na bagay

D. Sa paghahalintulad ng dalawang diwa

53. “Yumanig ang gusali sa kanyang mga yabag.” Ito’y isang ______________.

A. sinekdoke C. hyperbole

B. metafor D. personification
54. Ito ay mga butil ng karunungang hango sa karanasan ng mga matatanda at kina
papalooban ng mga paalaala tungkol sa batas ng mga karugalian. Ito ay isang _______.

A. salawikain C. kurido

B. bugtong D. awit

55. Ito ay nagbibigay din ng opinion tungkol sa isang napapanahong isyu sa pamamagitan ng
larawan. Alin ang tinutukoy?

A. Kartong pangeditoryal C. Larawan ng huling pahina

B. Ilustrasyon D. Larawan ng harap

56. Piliin sa sumusunod ang tumutukoy sa isang kilusang binuo ng pangkat ng mga intelektuwal
na humihingi ng reporma o pagbabago.

A. Katipunero C. Manggagawa

B. Estudyante D. Propaganda

57. Ito ay mga opisyal na wika mula sa panahon ng Republika hanggang sa kasalukuyan. Piliin
ito.

A. Filipino at Kastila C. Ingles at Kastila

B. Ingles at Tagalog D. Ingles at Pilipino

58. Kailan natin ipinagdiriwang ang lingo Ng Wika?

A. Marso 13-19 C. Agosto 13-19

B. Agosto 1-31 D. Hunyo 13-19

59. May pulong na gaganapin at isinulat na ang lugar, petsa at oras. Isinulat din ang “agenda”.
Ano ang maaaring salin nito?

A. Pagtatalunan C. Paghahandaan

B. Pag-uusapan D. Pagkakaisahan

60. Anong uri ng parirala ang sumusunod.

Ang matalinong mga bata ay nakakuha ng mataas na marka.

A. Pangalawang-diwa C. Pang-ukol

B. Panuring D. Pawatas

61. Pagtulungan at pagsasama-sama ang dapat gawin ng mamamayan?

A. Tambalan C. Karaniwan

B. Hindi karaniwan D. Payak

62. Ano ang karaniwang iisahang pantig lamang at walang katuturang maibibigay kung nag-
iisa?

A. Kataga C. Parirala

B. Sugnay D. Salita

63. Alin sa mga pangungusap ang walang paksa?


A. Agosto na talaga

B. Nag-aawitan ang nasa loob ng simbahan .

C. Dumating ka sana sa oras.

D. Ibinigay ko na ang pera sa iyo.

64. Ilan ang kasalukuyang bilang ng ating alpabeto?

A. 30 C. 20

B. 28 D. 31

65. Alin ang angkang pinagmulan ng mga wika sa Pilipinas?

A. Malay C. Indones Polinesyo

B. Indones D. Malayo Polinesyo

Review Materials For Professional Education


Multiple Choice:

1. An act known as the Philippines teacher’s Professionalization Act of 1994, to strengthen the
regulation and supervision of the practice of teaching is provided in _______________.

A. Republic Act No. 8736

B. Republic Act No. 7836


C. Republic Act No. 8980

D. Republic Act. No. 9880

2. “Learning has taken place when a strong bond between stimulus and response is formed.”
This is base on the theory of _____________.

A. constructivism

B. categorization

C. predisposition

D. connectionism

3. According to Ausubel, one of the ways to strengthen the students cognitive structure is by
using an instructional tool called ______________.

A. cross-referencing

B. spiral approach

C. advance organizer

D. narrative

4. Thorndike’s Law of effect states that a connection between stimulus and response is
strengthened when the consequence is ________________.

A. repeated

B. negative

C. pleasurable

D. positive

5. Watson applied classical conditioning in his experiments and the results showed that
behaviour is learned through stimulus-response associations, specifically the development of
emotional responses to certain stimuli. This help us in _____________.

A. understanding fears, phobias and love

B. interpreting reflexes as emotions

C. connecting observable behaviour to stimulus

D. understanding the role of overt behaviour.

6.Skinner is known for his theory based the notion that learning is a result of change in overt
behaviour, meaning, an individual responds to events that occur in the environment

A. operant conditioning

B. stimulus-response associations
C. classical conditioning

D. connectionism

7. The leading proponent of social learning theory is Bandura. He believes that _____________.

A. behavior can influence both the environment and the person.

B. learning stays with the individuals until needed.

C. reinforcement influences cognitive processes.

D. people learn from one another such as by modeling.

8. Should a teacher involve her / his students in planning the instructional objectives before
starting a lesson?

A. Yes, with shared objectives, they become self-motivated.

B. No, they will not be ready for it at the beginning

C. Yes, they will be given a tip on what questions to ask.

D. No, with cognitive objectives at the start, irrevelant questions are formulated.

9. Teacher A aims to develop critical and creative thinking among her students.

She should try using ____________.

A. declarative statements

B. convergent questions

C. divergent questions

D. expository statements

10. Lesson objectives must be aligned with the aims of education as embodied in the Philippine
Constitution and should be stated in the course syllabus.

What is a reason for this?

A. To learn more about the Constitution of our country

B. To develop mission-inspired citizens

C. To facilitate a faster and better learning

D. To instill the ideals and goals of the country

11. Read the dialogue

“Would you tell me which why I ought to go from here?” asked Alice.

“That depends a good deal on where you want to go.” said the cat.

“I don’t much care, where.” said Alice.

“Then it doesn’t matter which way you go” said the cat. Give the implication on instructional
objectives.

A. Objectives provide guides on questioning techniques.


B. With specific objectives, the lesson becomes more focused.
C. Objectives are open to students comments on the lesson.
D. With cognitive objectives, the affective domain is assured.

12. Instructional objectives in the affective domain includes receiving, responding, valuing and
the highest level is _____________.

A. acceptance or preference

B. selective attention and willingness

C. sequence or feelings of satisfaction

D. conceptualization of a value system

13. One of the principles of learning states that “learning is emotional as well as intellectual.”
Give your interpretation.

A. Internalization of ideas result to better learning.

B. Teachers should appeal their students acquisition of ideas.

C. Learning is maximized if feelings and thoughts are in harmony.

D. Teachers must draw the learner’s ideas and skills.

14. Miss Reyes observed that one of her students excels in activities requiring strength, speed,
flexibility, balance and hand-eye coordination. According to Howard Gardner, such natural
intelligence can be identified as _______________.

A. bodily-kinesthetic C. interpersonal

B. verbal-logical D. verbal-linguistic

15. Learning styles refer to the preferred way an individual processes information. Classify a
student who learns best through verbal lectures, discussions, talking things through listening to
what others have to say. He/ She is a/ an _____________.

A. visual learner C. analytic

B. auditory learner D. global

16. Identify the ninth intelligence which is described as the ability to seek connections involving
one’s place in the family, school, community and “role in the world.” They ask “why are we
here?” This intelligence is termed ___________.

A. naturalist C. intrapersonal

B. interpersonal D. existentialism

17. Children with sub average intelligence and deficits in adaptive behavior, experience difficulty
in managing daily activities and in conducting themselves appropriately in social situations are
diagnosed to be suffering from a kind of disability called _______________.

A. mental retardation C. emotional disorder

B. attention-deficit disorder D. health impairment

18. Difficulty in focusing and maintain attention coupled with recurrent hyperactive and
impulsive behavior are observed difficulties of children manifesting a kind of difficulty known

as ______________.
A. autism C. dyslexia

B. sensory impairment D. attention-deficit-hyperactivity disorder

19. Who is a major contributor in the development of the behaviorist theory?

A. Vygotsky C. Erikson

B. Pavlov D. Kohlberg

20. Erikson was influenced by Sigmund Freud and came up with a theory which serves as a
basis for analyzing personality and development to help facilitate the teacher’s understanding of
various environmental factors that affect own behavior and those of his/ her students”, as well.
This theory came to be known as _______________.

A. emotional development C. personality development

B. moral D. psycho-social

21. According to Dewey, teachers should make a significant analysis of their own actions,
decisions and successes or failures in teaching daily lessons. This activity is indicative of their
ability to empty ____________.

A. assessment agenda C. procedural learning

B. experiential teaching D. reflective teaching

22. What is one advantage of using the project method in science teaching?

A. It tests the student’s manipulative skill.

B. It requires students to present in concrete form how a principle works.

C. It requires assistance of experts on the subject.

D. It develop

23. It is an approach that makes students “think about their thinking,” Students get
consciousness of their thought processes while they are engaged in their cognitive tasks. This is
an example of an approach called ______________.

A. cognitive C. metacognitive

B. constructivist D. reflectivist

24. I like to show a close representation of the size and shape of the earth and its location in the
entire solar system. What is the best instructional aid?

A. picture C. realia

B. models D. films

25. The best way way to assess learning is to use real-life situations, objects and materials
existing in the environment. Hence, teachers are encouraged to use _____________.

A. rating scale C. observation technique

B. pencil-and-paper test D. authentic assessment

26. An appropriate assessment tool for assessing the development of learning in the affective
domain is through ________________ .
A. reading of journal entries C. product assessment

B. performance assessment D. self-assessment

27. The theme of Vygotsky’s socio-cultural theory emphasizes the role of appropriate assistance
given by the teacher to accomplish a task. Such help enables the child to move from the zone of
actual development to a zone of proximal development. Such assistance is termed _________ .

A. competency technique C. active participation

B. scaffolding D. collaboration

28. The heroism of Jose Rizal exemplifies Kohlberg’s theory of moral development. Which is
Rizal’s characteristic?

A. He possesses moral responsibility to make societal changes regardless of


consequences to oneself.

B. He believes laws that are wrong can be changed.

C. He gives importance to what people will think or say.

D. He is motivated to act by the benefit he gets later.

29. A left-brain dominant individual is portrayed as one who is analytic, linear and responds to
logic. A right-brain person is viewed as _______________ .

A. one who prefers to begin with details leading to a generalization

B. vernal, impulsive and learns sequentially

C. global, holistic and responds to emotions

D. one who is more comfortable with details and patterns

30. How can a teacher help students with different learning and thinking styles continue to learn
more effectively?

A. Allow sufficient time for processing different types of information.

B. Provide a general overview of the lesson.

C. Use a variety of reflection strategies.

D. Use questions of all types to stimulate various levels of thinking and valuing.

31. Students possessing intrapersonal intelligence, according to Gardner, learn through


dominant involvement of ______________ .

A. interaction with their environment

B. feelings, attitudes and values

C. reasoning and problem solving skills

D. interaction with others

32. A key concept in Tolman’s purposive behaviorism is “Learning is always purposive and goal-
directed.” He believed that individuals do more than respond to stimuli. This means that
__________ .

A. individuals responses are holistic and adaptive


B. they act on changing conditions, not merely respond to stimuli

C. there are variables that are determinants of behavior

D. reinforcement is not essential in learning

33. Ausubel recommends the use of visuals designed to bridge the gap between what the
learners already know and what they need to know. Which is an example?

A. Pictures C. Comparison patterns

B. Concept maps D. Overviews

34. The framework for creative thinking includes the production of a great number of ideas or
alternate solutions to a problem. Secondly, the ideas produced must show a variety of
possibilities and different points of views. Together they are considered effective in developing
creativity among students. What does the framework include?

A. Problem solving and enhancing C. Different approaches and strategies

B. Variety and strategies D. Fluency and flexibility

35. The process of problem solving and learning are highly unique and individual. This principle
means _______________.

A. students can adapt alternative problem solving models

B. students can modify their own personal styles

C. each student becomes aware of how learning styles can be changed

D. each student has his/ her own distinctive style of learning and solving problems

36. In the three level approach to teaching, values ate at the apex of the triangle. Now comes
the question, “Can values be taught?”

A. Yes, it is dependent on affective dimensions.

B. No, it is rather caught than taught.

C. Yes, it has cognitive dimension as well.

D. No, it cannot fit a subject matter content.

37. In the following method, the teacher starts a lesson stating the following findings:

The flies died after three days in a tightly-covered bottle.


The cockroaches caught in a covered tray died.
The rat caught in a deep hole was found dead.
Question: What caused the death of the animals?
Answer: Lack of oxygen
Generalization: Living things need oxygen in order to live.

What method of teaching is illustrated? Describe briefly.

A. Demonstration – present an activity to a group of observers.


B. Deductive – start with the generalization, then give examples.
C. Direct instruction – a step by step procedure is followed.
D. Inductive – from several examples, state a rule.
38. What is the best method to use in presenting a lesson which would need sophisticated and
expensive equipment and technical know-how?

A. Problem solving C. Demonstration

B. Inquiry approach D. Cooperative learning

39. Teacher B requested an older, brighter and more cooperative member in a group to teach
the other members.

A. The tutor is better equipped and close to the rest.

B. They know one another, learning is faster

C. The tutor is older and can command them to understand.

D. The members can rely on a brighter tutor.

40. Cooperative learning approach makes use of a classroom organization where students work
in teams to help each other learn. What mode of grouping can facilitate the skill and values
desired?

A. Whole class C. Heterogeneous

B. Homogeneous D. Competitive

41. It is a teaching approach that views learning as an active process that creates a
meaningful connection between prior experience and the present learning activity hence results
in better understandings and meanings. This approach is known as ________________.

A. inquiry C. problem solving

B. metacognitive D. constructivism

42. Research findings showed that student’s motivation may vary according to socio-economic
background. Which observation can attest to this?

A. Females are more likely than males to earn higher grades.

B. Students from low-income families are among those likely to be at risk for failing and
dropping from school.

C. Gifted students are more highly motivated


D. More boys than girls become underachievers

43. A teacher earns the title of “professional” if she/he is adjudged knowledgeable, proficient in
a variety of teaching methodologies and exhibits a high degree of third component of
competence which is/ are _________________.

A. mastery of content C. ethical and moral standards

B. a high degree of dexterity D. fairness and objectivity

44. It is a quality that is emitted like a “spark”. Dubbed as an energizer, it is evident in a


teacher’s eagerness and excitement in convincing children to get involved in the learning
activity. It is contagious. Name it.

A. Enthusiasm C. Compassion

B. Encouragement D. Interest
45. Teachers with integrity are perceived to possess inimitable probity, honesty in thoughts and
stature and indisputable fairness in making decisions.

Integrity can and should be mirrored. Is it sound and easy?

A. Yes, if the teacher is virtuous and principled

B. No, if she/ he is insincere.

C. Yes, if trust from parents is lacking

D. No, if consistently

46. “If is best to remember that when times are rough and difficulties arise, we have to stand
strong and remain committed to doing whatever is necessary to raise our children with
optimism, understanding and love”. This is a strong statement from a teacher processing a trait
called _____________.

A. patience C. industriousness

B. resilience D. sincerity

47. A school head envisions developing proficiency among her teachers in employing effective
and new teaching methodologies. What is one way of achieving it?

A. Attend annual conventions

B. Improvise of instructional tools

C. Conduct afterschool visits for exchanges of “best practices”, and observing class
demonstrations.

D. Undertake research in own specialization.

48. Ideal teachers view learning to teach as a lifelong process, hence strongly motivated to
continue growth as an individual and professional. Personal growth can be achieved through
one’s own initiative in __________________.

A. infusing values attitudes and desirable work ethics

B. interviewing model teachers and administrators

C. joining interest clubs and associations

D. attending training sessions and assemblies

49. According to Piaget’s stages of cognitive development, between ages 12 and 15 children
reach formal operational stage. What are they capable of doing at this stage?

A. Can focus on only one aspect of situation or event

B. Can solve abstract problems and think deductively

C. Reasoning is neither inductive nor deductive

D. Intelligence is intuitive in nature

50. In a Social Science class, Teacher M required the students to write the first and last stanza
of “Ang Pambansang Awit.” A student was asked how she was able to recall each word, “I had
sing it silently.” What is the implication of this regarding teaching strategies?

A. More active participation can make recall easier/


B. An integrated teaching approach is more effective in strengthening moral connections

C. Teaching strategies must make student’s experience concrete.

D. Teaching strategies can facilitate learning by appealing to their unique learning styles.

51. A teenage daughter was being checked by her mother regarding the hemline of her new
dress. It is very short, revealing half of her thigh. She argued, “Anne Curtis looks so pretty on
TV. My classmates and I decided to shorten our dress.” Does this show Bandura’s social
learning theory? If yes, state it, If no, why?

A. No, observes find it difficult to replicate someone’s behavior

B. Yes, behavior can be learned through modelling.

C. Yes, the person must pay attention to the model

D. No, observers may not be able to remember

52. The concept in Bandura’s social learning theory requires the following conditions to be
present for observers to learn from a model.

I. Attention III. Motor Reproduction

II. Retention IV. Reinforcement

A. I,II and III C. I,II and IV


B. I,II,IV D. II,III and IV

53. Gagne’s theory known as Conditions of Learning asserts that different internal and external
conditions are needed for each type of learning. What is an application of this theory?

A. Give a feedback for successful performance

B. To learn attitudes, the learners must be exposed to a credible role model

C. Success associated with the desired attitude must be known

D. Choose personal actions based on internal states of feeling

54. Lessons depicting historical events like declaration of independence on June 12 and Rizal’s
execution at Luneta can best be taught through which of these methodologies?

A. Discussion C. Team teaching

B. Role playing D. Narratives

55. In implementing an inquiry approach, recommend the most effective component to facilitate
the smooth probe of the concept to be learned.

A. Appropriate questioning techniques B. Different materials to be used

C. Lab manual D. Review of previous lesson

56. Field trip is an effective way of teaching where organisms could be observed in their natural
habitat, active and responding to stimuli in the environment. In addition, what values are
developed among the student’s during and after the trip?

A. Freedom in an open space than in a classroom

B. Love of nature, proper behavior during explorations


C. On-the-spot observation to keep them active

D. Consideration of the rights of others

57. Action research is a self-assessment procedure which teachers can undertake to gather
reliable information about their teaching practice in a particular learning context. The results can
assist them in appraising one’s own ________________.

A. classroom atmosphere

B. social interaction with their students

C. right and wrong perceptions

D. knowledge of teaching procedure

58. A teacher’s portfolio contains valuable information and evidence that can be analysed
regarding ______________.

A. problems encountered

B. relevant activities

C. new knowledge gained

D. mistakes corrected

59. Journals written by students at the completion of a day’s lesson will reveal which information
that can serve as guide in planning the next lesson?

I. Part of a lesson well understood or not

II. Topics most interesting

III. Difficulties experienced

IV. Emotional state throughout

A. I,II and III


B. I and III only
C. II and IV only
D. II only

60. When students recognize a situation as something similar in a way to what was experienced
before, the tendency is to use the knowledge and skills they learned to a new situation. This
theory illustrates _________________.

A. transfer of learning

B. information processing

C. degree of meaningfulness

D. meaningful learning

61. How’ can a teacher enhance her/ his questioning technique for an effective teacher-student
interactions?

A. Allow sufficient “think time,” at least 7-10 seconds

B. Extend wait-time until the student responds.


C. Immediately call another student in case one cannot answer

D. You may answer your own question if no one can

62. When asked why she has incomplete assignments everyday, she answered, “I have no one
to ask help. My parents leave for office very early in the morning and returns late, oftentimes
tired.” As her teacher what can you do?

A. Warn the parents that the child will fail if unaided

B. Meet the parents and together discuss how home and school can bridge their
concerns and assistance for them as learners

C. Write their parents about their responsibilities in helping their children as learners

D. Advise her to talk to her parents for much-needed help

63. You are to teach the difference between the parts of a complete and an incomplete flower.
They brought to class only common examples of complete flowers. What will you do?

A. Continue by reciting on the parts of both

B. Draw the parts of incomplete flowers if there is none

C. Go out to the lawn and look flowers for the weeds and wild grasses that are samples
of incomplete flowers

D. Emphasize the parts of a complete flower only.

64. Choose the important purposes of assessment.

I. Diagnosis III. Effectiveness of a program

II. Placement IV. Part of a lesson

A. I, II and III
B. I and II only
C. IV only
D. II, III and IV

65. At the preoperational stage of Piaget’s cognitive development, the child can see only his
point of view and assumes that everyone also has his same point of view. What is this tendency
called?

A. Transductive reasoning

B. Animism

C. Egocentrism

D. Conservatism

66. A child was shown an amount of water in a glass. The teacher poured the whole amount to
a much taller and narrower glass and marked this glass A. The same amount was poured in a
shorter and wider glass, marked glass B. When asked which has more water, A or B, the
answer was, “Glass B”. At what stage of cognitive development is it expected and what is this
ability called?

A. Concrete Operational stage; Conservation ability

B. Formal Operational stage; Deductive reasoning


C. Sensori-motor stage; Symbolic functions

D. Pre-Operational stage; Centration

67. In order to adapt to the students diverse background and learning styles, teachers may vary
their methods of assessing and evaluating student learning. Appropriate methods are:

I. Oral reports III. Dramatic presentation

II. Panel discussion IV. Construction projects

A. I, II and IV
B. I, II, III and IV
C. I and II only
D. I, II and III

68. Learning disabilities involve difficulties in specific cognitive processes like perception,
language, memory and metacognition and are not due to other disabilities like emotional and
learning disabilities like emotional and learning disabilities. Examples of learning difficulties are
_________________.

A. ADHD and Autism

B. Mental retardation and dyslexia

C. Dyslexia and dyscalculia

D. Speech and visual disorder

69. Who is the foremost believer that “intelligence is a function of the number of successful S-R
connection learned?

A. Thorndike

B. Watson

C. Vygotsky

D. Kohlberg

70. The International Commission on Education for the 21st Century chaired by Jacques Delors
advocates four pillars of education, namely:

I. Learning to know III. Learning to be

II. Learning to do IV. Learning to live together

Which are adopted today?

A. I and IV only
B. I, II only
C. II and III only
D. I, II, III and IV

71. An education for globalization should nurture the higher-order cognitive skill and
interpersonal skills in order to develop __________________.

A. student’s awareness of the world in which they live

B. ability to think creatively and ethically

C. ability to master ones national orogin


72. The teacher is the first audiovisual aid in the classroom. How can she act as one?

A. Sing before the class starts to keep them awake

B. Use TV and radio for every lessons

C. Have a pleasing physical appearance and modulated voice

D. Wear multicolored dress to catch their attention

73. The National College Entrance Examination which was then given by the DepEd for all 4th
year graduating students is a norm-referenced test. The purpose is to determine __________.

A. the scope or content coverage to be tested

B. the performance of a student with reference to a large group

C. who will construct the examination items

D. the performance of a student relative to a targeted skill

74. A criterion-referenced test is designed to determine whether or not a student has ________.

A. performed well in a wide content coverage

B. performed higher than the other members of a group

C. reached a present target

D. reached a performance level on a specific skill

75. How will you interpret a student’s 80% percentile score? The student scored ___________.

A. higher than 80% of the members of the group

B. better, relative to the competencies targeted


C. high in all the skills being tested

D. achieved 80% of the specified content

76. In planning a lesson, after establishing your objectives in the cognitive domain, the next
thing to do is ________________.

A. prepare a corresponding pretest and post test

B. find out what the students already know by giving a post test

C. get ready with all the materials that will be needed

D. find out which teaching methodology will be used

77. A teacher-made test given at the end of a lesson to find out if the objective has been
attained can be classified as ________________.

A. criterion-referenced

B. norm-referenced

C. placement test

D. curricular test
78. A teacher set 95% accuracy in a test on predicting the kind of weather given five different
atmospheric conditions. A student who obtains a score of 85% can be interpreted as ________.

A. she did not meet the set criterion by 10%

B. she is higher than 85% of the group

C. she obtained an 85% percentile score

D. she is 10% short of the set percentile score

79. Classify the type of the rest that is administered by the Professional Regulation Commission
aimed at measuring the proficiency of teachers in developing a set of instructional skills or
methodologies. The examination is given twice a year nationwide. The test is ____________.

A. norm-referenced

B. professional

C. criterion-referenced

D. performance test

80. A multiple-choice item consists of a stem representing the problem and the options
consisting of the list possible answers Example: During March, April and May, which of the
following seasons occurs? Options: Summer, Raining, Dry, Wet-and-dry. State a principle in
making options for this particular test.

A. The correct option must not stand out from the category

B. The wrong options must be seasons also

C. Include only one possible answer

D. The stem must not provide a clue

81. Avoid negative statements in the stem of a multiple-choice stem. Such as “It is not true that
Rizal died on December 30.” Why?

A. It is open to guessing

B. It is confusing

C. It is only single recall

D. It is memory learning

82. A major advantage of an essay type of test is its ease in measuring the student’s abilities
to ____________.

A. present a wide range of information

B. suggest innovations in solving a problem

C. organize and synthesize own knowledge

D. include plain guesses in a nice way

83. What are the characteristics of a standardized test?

A. High reliability index


B. Mostly criterion-referenced

C. Usually administrated to small groups

84. A college administrator wanted an entrance examination that can identify future outcome or
differences such as showing who will graduate from college or who will drop out. Such a test is
one that has _____________.

A. norm-related validity

B. predictive validity

C. construct validity

D. concurrent validity

85. Which are attributes of a good evaluation instrument?

I. Validity III. Sensitivity

II. Reliability IV. Objectivity

A. I, II and IV
B. I and II only
C. I, II and III
D. I,II,III and IV

86. A table of Specifications presents the details of the following areas:

a) number of items, b) percentage of items in each area to be tested and

c)________________________________.

A. objectives according to bloom’s taxonomy

B. type of instrument you will construct

C. scoring procedure

D. content validation

87. One of the characteristics of a good test is content validity. What is the first thing to be done
before test construction?

A. Find out if the items are constructed well

B. Judge the scope or content coverage.

C. Find out if the number of items is sufficient

D. Examine the objectives set for the test to find out it they are consistent with the
subject matter.

88. Item analysis is undertaken for the purpose of determining the difficulty of the item and the
discriminating power of the item. The ensure an adequate sampling a teacher may

prepare a _____________.

A. List of concepts to be tested


B. Table of Specifications
C. Number of objectives
D. Number of items
89. How can a teacher establish the reliability test?

I. Repeat the same test. III. Split the test

II. Administer a parallel test IV. Vary the number of items

A. I, II and III
B. I,III and IV
C. I and IV only
D. II and IV only

90. In measuring student’s development in the affective domain, the hierarchy or stages
according to Bloom’s taxonomy occur in this sequence:

A. Awareness, Appreciation, Acceptance and Valuing

B. Appreciation, Awareness, Acceptance and Valuing

C. Valuing, Awareness Acceptance and Appreciation

D. Acceptance, Awareness, Appreciation and Valuing

91. Which is the most reliable and easily-used attitudinal instrument for measuring the most
positive and favourable to the most negative and unfavourable views about a concept?

A. Temperament Survey

B. Semantic Differential

C. Likert Scale

D. Adjustment Scale

92. In the accreditation program being required by the CHED in assessing the quality of
instruction in universities and colleges, which among the set of criteria that must be included in
determining the set levels.

I. Mission and vision of the school III. Curriculum

II. Faculty profile IV. Building and facilitates

A. I, II, III and IV


B. I, II and III
C. I and IV only
D. II and III only

93. “The process of democratic consultation shall be observed in the decision-making process
at appropriate levels.” This principle of shared governance is provided in
Republic Act No. ____________________.

A. 7836 C. 8980

B. 9155 D. 7610

94. On which policy is R.A No. 4670 known as the Magma Carta for Public School Teachers
focused?

A. Right to establish or join organizations

B. Code of ethics for professional teachers

C. Recruitment shall take place after training


D. To promote and improve the social the economics status of public school teachers

95. According to the 1987 Constitution, when should religion be taught to children in public
elementary and high schools?

A. During Fridays C. During Saturdays

B. After the regular class hours D. Within the regular class hours

96. Section 6 of the 1987 Constitution states that the government shall take the steps to initiate
and sustain the use of Filipino as a medium of official communication and as language of
instruction. The official languages are ________________ .

A. Filipino and Spanish C. Filipino only

B. Filipino and English D. Arabic

97. The Bureau of Nonformal Education which provides quality basic education for out-of-school
youth and adult learners has been renamed _____________ .

A. Bureau of Alternative Learning System C. Learners Outside the School System

B. Bureau of I nformal Learning System D. Bureau of Formal Education System

98. The PRC Code of Ethics for Professional Teachers states that every teacher shall pursue
studies that will improve his efficiency, shall strengthen his competence, virtues and productivity
and enhance the prestige of the profession. This refers to ___________ .

A. Standards of Quality Education

B. Trusteeship of Teachers Agenda

C. Continuing Professional Education Program (CPE)

D. Advancement of Teachers Program

99. In order to arrive at a skills-based evaluation, what could insure an objective higher-order
assessment?

A. Analyze the different answers C. Use of a scoring rubric

B. Assess the product only D. Use self-assessment

100. The Educational Media Center (EMC) is a vital instrument for quality education processes:
it is fully equipped to serve as ________________ .

I. a learning laboratory III. coordinating agency

II. service agency IV. center for recreational reading

A. I, II, III, IV C. I and IV only


B. I, II and III D. I and II only

101. Who authored the three-tiered model of learning-enactive, iconic and symbolic and
emphasized that we should not teach directly to the abstract level without adequate foundation
of the concrete?

A. Jerome S. Bruner C. Erik Erikson

B. Jean Piaget D. Leo Vygotsky


102. Teacher B wanted her students to learn the electoral process in action. So they went on to
elect the Presiddent, Vice President, 12 Senators and 20 Congressmen. They came up with
qualification of candidates, manner of campaigning and canvassing of votes.

This activity is described as a copy of real event which can simulate real-life situations
the students learned from _____________ .

A. contrived experiences C. purposeful experience


B. direct experiences D. demonstration

103. The use of planetarium to show the arrangement of planets, together with their rotation and
revolution around the sun is an example of a teaching strategy called ___________ .

A. demonstration B. use of models

B. simulation D. direct instruction

104. Teacher E interviewed five students for their reasons for being absent twice during the
week for several months. Then she showed the whole situation in a cause (lack of transport
fare) –and –effect (missing a day in class) diagram by the way of a ___________ .

A. fishbone diagram C. flow chart

B. tree diagram D. stream chart

105. The use of TV and film bring to the classroom “live broadcasts” of world events as they
happen. What are other advantages of using it. The students are provided _____________ .

I. exciting sights and sounds

II. close up view of big events

III. most believable news source

A. I, II and III C. I and III only


B. I and II only D. II and III only

106. Using TV and films has limitations also as instructional tools. As teacher, which is missed
by the students during the viewing?

A. The small screen size

B. The meaning of the lesson objectives

C. the chance to interact and be creative

D. hands-on activities

107. As an insightful teacher, how will you advise the parents about the far reaching effects of
media on the young minds?

A. Choose educational TV programs to help them learn.

B. Let the children choose those they enjoy.

C. Choose their own time in viewing.

D. Tell them that high exposure to media has advantages.

108. In a project-based multimedia learning strategy, the students learn new knowledge and
basic skills. Aside from being actively engaged in the learning task, they get involved in
_________ .
A. acquiring technological skills rather than content

B. making use of extended period of time allowed

C. learning to use the computer in organizing data

D. planning, designing and producing a multimedia product for presentation

109. As a teacher employing the project-based multimedia learning strategy, what are some
limitations you expect from the encounter?

I. There is a need for extending the time to use several media.

II. The presentation of the product is not an easy task.

III. The technology skills to produce a product may be lacking.

A. I only C. III only


B. I, II and III D. I and II only

110. According to Jonathan Herbart, which of the following are the formal steps of instruction
arranged in their proper sequence?

A. Preparation, presentation, association, generalization

B. Presentation, preparation, association, generalization

C. Generalization, presentation, preparation, association

D. Association, preparation, presentation, association

111. The 19th century saw the development of state-controlled and state-supported public
school system. Existing ideology was the concept of national sovereignty. Under a democracy,
education equipped citizens with the principles and duties of citizenship to guarantee national
stability.

Such is the ideology of _________________ .

A. citizenship C. nationalism
B. progressivism D. secular education

112. Rousseau established a number of modern principles of teaching aimed at the natural
goodness and virtue of man. Which of the following principles were his great contributions?

I. Principles of Growth

II. Individualization

III. Principles of Pupil Activity

A. I, II and III C. II and III


B. I only D. I and III

113. A report from PAGASA showed an increasing trend in daily temperature from January to
March. How can students best present such trend to the class?

A. Flowchart C. Drawing

B. Map D. Bar Graph


114. A town mayor was requested to present how the different offices in the municipal hall are
related to one another according to their related functions. Which one should he prepare?

A. Line graph C. Time chart

B. Organizational chart D. Stream flow

115. In a Biology class, the students were asked to trace their own families’ closest relatives
starting from their grandparents. They constructed a starting couple and added several
branches to indicate the offsprings up to 2 or 3 generations. Which did they construct?

A. Pictorial graph C. Fishbone diagram

B. Family tree D. Affinity diagram

116. In Social Science, which is the best visual in identifying the prominent cities and provinces
located in the different regions of the country?

A. Political map C. Relief map

B. Economic map D. Physical map

117. Which philosophy proclaims the “spiritual nature of man” and stresses that the human
spirit, soul or mind are the most important elements in life and “that the true, good and beautiful
are permanently part of the universe”?

A. Essentialism C. Perennialism

B. Realism D. Idealism

118. Which theory states that the significant skills, knowledge and attitudes in making one’s
adjustment to the realities of life should be systematically planned. Which thing emphasizes the
authority of the teachers and the value of a subject matter curriculum. It emphasizes habituation
more than experience, discipline more than freedom. It flavors drill in developing skills.

A. Essentialism C. Perennialism

B. Idealism D. Realism

119. “All learning should center on the child’s interests and needs.” The school should be a
pleasant place for learning. Its emphasizes is on the child as the learner and not on subject
matter and stress on activities and experience rather than on textbook.

A. Progressivism C. Perennialism

B. Reconstructionism D. Pragmatism

120. Which interesting group of theorists believe that the human being is the creator of his own
essence; he creates his own values through freedom of choice or preference. Education is
developing awareness about responsibility for one’s choice.

A. Reconstructionists C. Individualists

B. Existentialists D. Humanists

121. The Centers for Excellence in Teacher Education in the Philippines was created

by _____________.

A. Republic Act No. 7784 C. Republic Act No. 7791


B. Republic Act No. 7836 D. Republic Act No. 9917
122. What are the sources of one’s educational philosophy?

I. People and kinds of relationship

II. School and teachers in them

III. Sociocultural environment

A. I, II and III C. II and III only


B. I and II only D. I and III only

123. The cycle in curriculum planning constitutes three fundamental elements.

First concern are the goals and objectives, followed by the body of learning experiences and
resources (methodologies) and ends with a/ an _________________.

A. revision of methods C. assessment of outcomes


B. reconsideration of the goals D. orientation for next activities

124. Spartan education provided so much attention and time for the art of war and the training
of soldier-citizens. Hence the Spartan curricula consisted of military exercises in services for
the state.

Athenian education, on the other hand, stressed individual excellence, hence


aimed to provide _______________.

A. Moral training, emphasized virtues, to develop personality


B. Learners with training in harsh discipline
C. full-rounded development to fight as soldiers
D. control of training of children

125. Comenius, a prominent sense realist, believed that the aim of education is “to know all
things, to do all things, and to say all things”. He favoured a democratic education and the use
of the vernacular. He believed that the best agency for education is the ________________.

A. textbooks C. government

B. school D. master

126. During the age of naturalism, the duty of parents in the education of the child was
emphasized. Parents should assist in the natural unfolding of the child to be free to develop
according to his own natural impulses, “free from the artificialities of human society.” The
famous naturalist is ___________________.

A. Jean J. Rousseau C. Mulcastar

B. Francis Bacon D. Charlemagne

127. Friedrick Froebel strongly believed that the aim of education was the development of the
child. The teacher should be a moral and cultural model worthy of the child’s love and trust.
Froebel is known for his __________________.

A. idea of justice C. idea of retribution

B. kindergarden D. democratic education

128. He is known for his philosophy of pragmatism and synthesis of Darwin’s theory of
evolution. He believed that children were sociality active human beings and they want to explore
their environment, He favoured democratic education. Who is referred to?

A. Pestalozzi C. John Dewey


B. Jonathan Herbart D. Thorndike

129. A negative discrimination index means ____________________.

A. more from the lower group answered the test item correctly

B. more from the upper group answered the test correctly

C. the test item could not discriminate between the upper and the lower group

D. the test item has low reliability

130. What can be said of student performance in a positively skewed score distribution?

A. Most students performed poorly

B. A few students performed excellently

C. Almost all students had average performance

D. Most students performed well

131. A teacher would use a standardized test in order to ____________________.

A. serve as a final examination

B. compare her students to national norms

C. engage in easy scoring

D. serve as a unit test

132. The main purpose in administrating a pre-test and a post-test to students is to _________.

A. accustom the students in frequent testing

B. keep adequate records

C. measure the value of the material used

D. measure gains in learning

133. A number of test items in attest are said to be nondiscriminating. What conclusions can be
drawn?

I. Teaching or learning was very good

II. The item is so easy that anyone could get it right

III. The item is so difficult that nobody could get it

A. II and III
B. I only
C. III only
D. I and II

134. A test item has a difficulty index .51 and a discriminating index of .25. What should the
teacher do?

A. Retain the item

B. Make it a bonus item


C. Revise the item

D. Reject the item

135. The difficulty index of a test is 1. This means the test is ________________.

A. a quality item

B. very difficult

C. very easy

D. missed by everybody

136. If the computed range is low, this means ___________________.

A. the difference between the highest and the lowest score is low

B. the students performed very well in the test

C. the difference between the highest and the lowest score is high

D. the students performed very poorly in the test


137. Which groups of scores is most varied? The group with _________________.

A. .90 SD C. .10 SD

B. .50 SD D. .75 SD
138. What does a percentile rank of 72 mean?

A. The student’s score is higher than 72% percent of all students who took the test

B. the student answered 72 percent of the items correctly

C. 72 percent of those who took the test scored higher than the student

D. It is the student’s score in the test

139. What is the mean of this score distribution: 30, 31, 25, 28, 29, 27, 33, 35, 28? __________.

A. 29.00 C. 28.55

B. 29.55 D. 28

140. What is the median of the following scores: 45, 42, 40, 39, 41, 37, 36, 40.

A. 40 C. 38

B. 39 D. 41

141. A portfolio assessment requires a presentation of a collection of student’s work. For which
purpose/s is it created?

I. To show growth III. To evaluate

II. To showcase current abilities

A. I, II and III C. I and III


B. I and II D. II and III

142. A teacher is following the average grading system. In four grading periods, a student
obtained 80, 82, 84 and 85. What is her average?
A. 82 C. 83

B. 82.75 D. 83.5

143. Teacher M was observed by her Subject Coordinator to be using most often the
questioning and answer approach. This method was popularized by _____________.

A. Schmucks C. Berliner

B. Socrates D. Solomon

144. What does a skewed score distribution mean?

A. Scores are concentrated more at one end or the other end of the normal curve.

B. Most of the scores are on the lower end of the curve.

C. Most of the scores are on the upper

D. The scores are normally distributed

145. How many percent of the cases fall between -1 and +1 SD units from the mean?

A. 68% C. 95%

B. 38% D. 99%

146. An examinee whose score is within X + 1 SD belongs to which of the following groups?

A. Below average C. Average

B. Above average D. Needs improvement

147. One interesting instructional visual is Edgar Dale’s Cone of Experience. Visualizing the
cone which represents his theory on the importance of sensory experiences, what are at the
bottom and at the top of the cone?

A. Direct purposeful experience experience; Verbal symbola

B. Contrived experiences; visual symbols

C. Demostration; motion pictures

D. Dramatized experience; study trips

148. Edgar Dale’s Cone of Experience implies that first hand experience is the foundation of the
student’s learning. This means that we _________________.

A. learn the then proceed to the concrete

B. must start with the concrete and progress to the abstract.

C. must end with the abstract

D. get stuck with the concrete without moving to the abstact

149. Robert Gagne, like Edgar Dale, specified 9 different levels of learning. He stressed that
different internal and external conditions are needed for each type of learning. Likewise those
levels must be sequenced or arranged in the proper order. Gagne’s theory is referred to
as _______________.

A. categories of learning C. designing instruction


B. instructional events D. conditions of learning

150. Theorists believe that learners should be allowed to discover principles through their own
explorations rather than direct instruction by the teachers. This view is called _____________.

A. knowledge construction

B. social constructivism

C. individual constructivism

D. social context of knowledge

151. Which is one very important characteristic of a constructivist teacher?

A. Plays the role of a facilitator rather than an expert who has all the knowledge

B. Capable of employing direct instruction

C. Presents her own view for the students to start theirs

D. Emphasizes that knowledge exists in a social context

152. At Piaget’s pre operational stage ( 2-7 years) children possess the ability to represent
objects and events. Drawings, written words, and spoken word comes to be understood as
representing a real object like NOKIA cellphone. Girls can pretend to host a grand party which
exist only in their minds. They can now make mental representations and can pretend. The
stage is termed ________________.

A. Representing through symbols

B. Concrete interconnection

C. Analogical reasoning

D. Stage of animism

153. According to surveys, one of teacher’s affective traits is caring, Which describes one who
exhibits naturally this attribute?

A. Showing respect for students on social occasions

B. Willing to work with them in completing learning tasks

C. Behaving friendly consistently

D. Sympathetic concerns about their student’s lives, listening and understanding their
personal joys and sadness

154. Parents are an essential part of their children’s human learning environment. Which
behavioural characteristics of parents support their children’s learning?

I. Follow status of children’s performance

II. Participate ins school community projects

III. Ask teacher why son’s grade is too low

IV. Supervise their children’s homework

Which common activity would you recommend to parents?

A. I, II, III, IV C. I and III


B. I and IV D. II, III and IV

155. Constructing a project at the ned of a unit is used as an assessment strategy. Which is an
advantage?

A. Students are encouraged to show their innovation

B. Parents will be able to do it for their children

C. It develops students’ manipulative skill

D. Student present a principle learned in concrete form

156. In a lesson on kinds of leaves, which is the best teaching strategy and why?

A. Research; develops the ability to widen the search

B. Constructing a project; develops skill in designing

C. Special report; able to find information about rare kinds

D. Preparing a collection of leaves; develops skill in classifying, and organizing the


materials learned

157. Which one describes journal writing as a teaching technique?

A. It is record of their attitudes, habits of work in one day.

B. It is like a diary of what transpires during a learning episode

C. It is a good source of information about one’s own strength and weaknesses

D. It serves as an evaluation instrument

158. Teacher A was asked to implement an individual teaching strategy, also called a self-
learning technique. Which one is an individualized teaching strategy and which benefit is
derived from its use?

A. Special reports; enables student to do actual observations in the environment

B. Child study; provides time for student work alone

C. Project method; tests student’s creativity

D. Independent study; enables student to gain insights on “how to learn” on this own

159. According to Ausubel, the students cognitive structure can be strengthened by allowing
them to have to a bird’s eye view ot to see “big picture” of the topic to be learned even going to
the details. This is achieved by ________________.

A. presenting overviews

B. cognitive structure recall

C. preparing summaries

D. using advance organizers

160. “What is learned is based on what is already known”. This implies that prior knowledge and
biases limit and affect what is learned. This is known as Ausubel’s ______________ .

A. subsumption theory C. cross-referencing


B. learning by skimming D. making comparisons

161. Research shows that females are more likely than males to have a higher need for
affiliation and that more boys than girls end up as underachievers. What factor accounts for the
difference in their motivation?

A. age C. cultural background

B. gender D. socio economic status

162. “Students from low-income families are among those most likely to be at risk for failing and
dropping out of school.” Which is referred to as the cause of student’s poor performance?

I. Distance from school

II. Insufficient financial support

III. Unhealthful condition

IV. Lack of early experiences

A. II and III C. I and II


B. I and IV D. I, II and IV

163. Read this letter from a former student, then answer the question.

“Thank you very much for the patience you had with me and for sharing your precious
time. . . . You are such a wonderful person, very down to earth, with hands always open to help.
. . . I am truly grateful for your loving kindness. You were our model. We hope to be like you
when we face our students in the future.”

Behavior can be learned through modelling, according to _____________ .

A. Jerome Bruner C. John Watson


B. Wolfgang Kohler D. Albert Bandura

164. Danielson’s (2002) advice is “the bell curve mentality must be abandoned.” Why?

A. It is possible that all students will pass the test.

B. Students will not understand the meaning of the curve.

C. The poor students are expected to fail.

D. It is normal that some will pass.

165. How can a teacher avoid “breakdowns and interruptions” in daily class procedures?

A. Assign a leader to assist everyone.

B. Establish routine for daily tasks.

C. Punish the misbehaving student.

D. Allow the students to make their own regulations.

166. “Have eyes at the back of your heads.” What does this reminder suggest regarding
discipline during the lesson?

A. Pupils will behave because we know what they are doing

B. Looking at our back will keep them interested


C. Our back part can also discipline them

D. We can easily turn our back to the front

167. Cite some teacher’s personal attributes that could prevent common classroom discipline
problems?

A. Passion and commitment

B. Patience and compassion

C. Ability to ridicule and embarrass

D. Tendency to give nag and to give sermons

168. Tolman’s concept showed a kind of learning that stays with the individual “until needed.” It
is not outwardly manifested at once. Through our learning gained beforehand, we will be able to
recall it when needed. This is a kind of learning called ____________.

A. latent C. mapping

B. active D. random

169. When we use acronyms to help us retain and retrieve information more effectively, such
techniques use ________________.

A. part learning C. mnemonic aids

B. massed practice D. recency list

170. A Physical Education teacher is teaching a popular native dance. She described each step
and demonstrated step by step from the start to the fith. Then she asked the group to execute
the steps in the same order.

What instructional approach did she use?

A. Deductive
B. Direct Instruction
C. Constructivist
D. Inductive

171. Which describes the constructivist view of learning?

A. Student’s own stored learning experience is easily recalled

B. Reflecting accumulated knowledge is the core of learning

C. Learning can be achieved through fixed documents and laws

D. Learning results by establishing a “meaningful connection” between prior knowledge


and the present learning activity

172. Which approach makes students “think about their own thinking?

A. self-directed C. reconstructive

B. reflective D. metacognitive

173. When students get actively involved in the construction and reconstruction of their own
meanings and understandings, it results in a uniquely __________________.

A. in-depth investigation
B. self-discovered

C. innovative thinking

D. analytic information

174. Which is a very effective approach where students work in groups to help each other learn.
In small groups they solve their onw problems and gain information through collective effort.
Reward systems are group-oriented and is used with heterogeneous grouping?

A. Cooperative learning approach

B. Partner learning

C. Problem solving approach

D. Tutorial approach

175. Experiencing success in discovering lessons builds up the children’s feelings of


confidence. Then, they would want to do more and discover more. This is
achieved through _______________.

A. inquiry approach

B. demonstrations

C. discussion

D. investigation

176. What values are developed by the discovery approach?

A. Responsibility and originality

B. Cooperation and sharing

C. Open-mindedness

D. Skill in formulating hypothesis

177. Disabled students are now better addressed as ________________.

A. suffering from

B. physically challenged

C. with exceptionalities

D. needing assistance

178. Information Processing Theory illustrates some methods for increasing retrieval of
information. Which are the methods?

I. Rehearsal- repeating information verbatim

II. Making connections between new information and prior knowledge

III. Elaboration or adding new information to the old

IV. Making the information relevant to the individual

A. I, II, III, IV
B. I, III and IV
C. I, II and III
D. II, III and IV

179.When certain topics are presented in the lementary in a manner appropriate for grade
schoolers, and the same topic is tackled in the high school, but on a much deeper level, this is
referred to as a ________________.

A. discovery learning

B. classified curriculum

C. spiral curriculum

D. symbolic learning

180. Which guidelines should be followed to ensure the effective use of the discovery approach.

A. Do not accept trial and error attempts

B. Make sure all instructional materials are available

C. Assign them the plan of attacking their own problem

D. Let students find answers to problems by themselves

181. Which is an advantage and a disadvantage of using real objects in teaching?

A. Real objects are easily available; some real objects are potential hazard.

B. Real objects can be observed thoroughly; some are expensive.

C. Students are interested with real objects; may get destroyed easily.

D. Can fit real objects to the lesson; children forget to bring the right kind.

182. A student observed that her teacher is wearing big earrings, three bracelets and two rings.
Immediately, she told her classmates, “I think she is very rich.” This statement is an example of
a/ an _________________.

A. observation

B. conclusion

C. inference

D. generalization

183. A teacher gave a 10-item test after the lesson. At the end of the unit she prepared a long
test to find out if the unit was successfully learned. Which kinds of tests were administered
during and after the unit.

A. Formative; summative

B. Beginning; final

C. Preparatory; summarizing

D. Diagnostic; final

184. A test was composed of test items that were declared to be nondiscriminating. What could
be the reasons for such conclusion?

I. Teaching and learning was good


II. The items are so easy that anyone could get them right

III. The items are so difficult that nobody could pass them

A. II and III
B. III only
C. I and II
D. II only

185. A class in the Bicol region was brought Mt. Mayon to view it at a distance. Upon their return
the teacher asked them to present to other classes what they learned in the most concrete way.
Which presentation is most life like?

A. A group molded clay to form a cone with an opening on top

B. A group presented cut outs of a volcano from the newspaper

C. A group made a large drawing on Manila paper

D. A group described orally what they saw

186. Learning hierarchies, that is “knowing the prerequisites that should be completed according
to complexity,” provides a basis for sequencing instruction. This is an important
recommendation of ___________________.

A. Gagne C. Ausubel

B. Thorndike D. Bruner

187. Transfer of Learning happens when learning in one context affects performance in another
similar situation. Which principles are based on the factors that affect transfer of learning?

I. Similarity between two learning situations

II. Degree of relevance of Learning

III. Emphasis on metacognition

A. I, II, III C. II and III


B. I only D. I and II

188. Everyone learns from an experience but experience alone is not sufficient for effective
learning. It is refection that enables the students to analyse and evaluate their experience.
Which strategy can make students benefit from their reflections?

A. Collecting photographs C. Preparing a Diary

B. Constructing an album D. Keeping a portfolio

189. Which physical learning environment is conducive to learning?

I. Allows student movement

II. Safe, clean and orderly

III. Well-ventilated

IV. Adequately lighted

A. I, II, III, IV C. III and IV


B. II, III, IV D. II only
190. Teacher C emphasized that one of the purposes of assessment is to develop the capacity
of her students for self-assessment. How will she do self-assessment?
Allow them to ______________.

A. reflect on their own personal characteristics that affect their learning

B. ask classmates to evaluate their own performance according to their experience

C. plan the tasks by themselves without asking help

D. take an active role in their own learning and assessment

191. A teacher who has been teaching for the past 20 years refuses to attend seminars. She
reasons out that she is knowledge enough. What is she missing by such reaction?

A. Knowing some more knowledgeable teachers

B. Updating through continuing professional education

C. The importance of being knowledgeable

D. Updating through reading about the latest

192. Teacher A asked her students to enact a lesson dealing with water conservation.
Immediately the class formed two groups; one with the mayor talking to the town folks, another
with a mother asking her children how they can do it. Which teaching method did she use? Is it
appropriate for solving the problem on conservation?

A. Cooperative learning approach; Yes, a group work develops the spirit of cooperation

B. Discussion; No, the mayors and the mothers role are difficult

C. Reporting; Yes, water conservation in the home is easy to enact

D. Inquiry Approach; No, water conservation in the community is difficult to enact

193. In a course on Methods of Teaching, the professor asked, As future teachers, what
teaching strategy to enable young children to “learn how to learn” will you use? Defend your
choice.

A. Direct instruction; There will become independent learners

B. Inquiry/ discovery. They will become independent learners

C. Demonstration, It is easy for them to follow

D. Discussion; She can discover how they learn.

194. A teacher wants her students to study things in their natural setting. Nature and organisms
are best learned by on-the-spot observations rather than discussions in the classroom.

Which teaching methodology is most effective?

A. Film viewing C. Travelling


B. Conducting D. Putting up an aquarium

195. Which is an advantage of presenting lessons through education channels on television?

A. Places, events and objects projected on screen help in visualizing life-like happenings

B. They can view the events over and over again, especially current events
C. The lesson viewed is the same for all viewers and the reactions will be the same

D. It does not need so much activity around because they are assigned seats.

196. Who are considered community-based stakeholders in curriculum development?

A. Parents, guardians and professional groups

B. School personnel and teachers

C. Students and government officials

D. Textbook publishers

197. Who are the school-based stakeholders in curriculum development?

A. Parents, guardians and professional groups

B. School personnel and teachers

C. Students and government officials

D. Textbook publishers

198. Which steps are included among Hilda Taba’s Inverted Model of Curriculum Development?

I. Diagnosing needs

II. Formulating specific objectives

III. Selecting content

IV. Determining school’s philosophy

A. I, II, III C. II, III, IV


B. I, III, IV D. I, II, IV

199. Identifying educational purposes and determining the school’s and teacher’s philosophy
are steps followed in an influential curriculum model identified as ________________.

A. ends-means C. traditional

B. inverted D. non-traditional

200. In organizing and designing a curriculum, what would selection of content or subject matter
include?

I. Knowledge

II. Skills

III. Interest and Learners

IV. Values and attitudes

A. I, II, IV C. II and III only


B. I and IV only D. I and III only

201. Preparing the students to think critically and ensuring that democratic principles are carried
out are considered important in developing a curriculum. This is in accordance with the
philosophical baliefs of the _______________.

A. nationalists C. constructivists
B. idealists D. pragmatists

202. In evaluating a curriculum, which is the most valid criterion to observe?

A. Methods C. Activities

B. Available resources D. Consistency with objectives

203. How can a teacher develop the value of timeliness such as punctuality and maximal
utilization of time?

A. Consistently follow schedule for classroom routine

B. Rush if you are getting late for the right time

C. Avoid disruptions due to improper behavior

D. Have a big clock be installed in the classroom for everyones guidance.

204. Discipline is an important concern of teachers to attain good management. Which is a


common cause of discipline-challenging situations which could be traced to the teacher herself?

A. Personal and emotional attributes

B. Lack of skill in scheduling time for activities

C. Inability to make advanced preparations

D. Lack of time-consciousness

205. Of prime importance in coping with discipline problems are the teacher’s personal
attributes such as,

I. Patience III. Concern and caring attitude

II. Compassion IV. Respect and trust for others

Which qualities are proven by research to be effective in preventing discipline problems?

A. I, II, III, IV C. II, III, IV


B. I, II, III D. I, III, IV

206. Today’s classroom are filled with 60 to 70 8-year olds. What unfavorable learning situation
can result?

I. Insufficient instructional materials

II. Louder noise from a big group

III. Overcrowding can cause discomfort

IV. Immobility and inappropriate seating arrangement

A. I, II, III, IV C. I and II


B. I, II and III D. II and III

207. Is “discipline the exclusive responsibility of the teacher?

A. No, students must likewise be allowed to participate in formulating rules for their own
behavior

B. Yes, it is their own sole authority to discipline them and not the students
C. No, some teachers lack the skill in preventing misbehaviour, hence misbehaviour
arise

D. Yes, proper behavior can favour learning situation and the skill

208. “Assertive discipline” is exercised by those who believe that discipline is the right of the
teacher. How would you characterize a classroom where students are duty-bound to follow rules
strictly? Such a classroom is described as ___________________.

A. democratic C. authority-controlled

B. autocratic D. well-managed

209. “Time lost is irretrievably,” said Jose Rizal. What is the implication of this famous truism
when applied to classroom management of time?

A. Teachers must be ready for any time interruptions due to poor time schedule.

B. Teachers can make up for lost time anyway, they have not to worry.

C. Allow flexibility for scheduling time avoid strict compliance

D. Teachers should maximize the time for instruction and avoid haste and waste

210. Teacher A has a monthly calendar tacked on a wall. Side by side she places a day’s hourly
schedule of activities without fail. Everyday the students get important
information about _________________.

I. length of time to be spent for each activity

II. sequencing of the activities from beginning to end of class

III. the day of the month they are in

A. I, II, III C. I and III only


B. I and II only D. II and III only

211. The class finished the experiment including formulating their conclusions. There is still time
left. What can she do to make use of the time left?

A. Use fillers, like writing a journal about how they feel about their new discovery/
learning.

B. Give a test to find out if their conclusion is correct and let them check it

C. Send them to library for extended readings and require a special report

D. Give another experiment related to what they finished and allow same time.

212. Teacher C noticed that her students often get so interested with an activity that they go
beyond the time allotted for it. Should she cut short their continued observations and order them
to proceed to the next activity?

A. Avoid rushing them, you should have allotted sufficient time for them to spend

B. You don’t have to be strict with the allotted time especially that you can see how
eager they are to stay

C. No extension of time, that’s all, then proceed to the next activity

D. Allow just a little more time and warn that next time they have to follow the scheduled
time allotment
213. Supposing despite early preparations for smooth transition from one activity to another, an
unexpected failure of operation of an instructional device like a projector or a computer
happens. Could the teacher be blamed for the idle time that ensues? What should she have
done?

A. Yes, anticipate a failure and get ready with an alternative plan

B. No, she is confident the devices are in good condition, no need for a plan

C. Yes, she should know how to repair any failures or ask for help immediately

D. No, she can continue the activity the following day rather than wait

214. On the first day of school, Teacher E announced to the class procedures to be observed in
passing papers, getting laboratory materials and returning them on the shelves, and passing in
and out of the room when needed. The chairs are arranged to provide mobility and space.
Routine activities are agreed upon. Which describes teacher E’s approach to discipline?

A. Proactive C. Dialogical

B. Reactive D. Routinary

215. Why must teachers be alert and consistent in classroom management?

A. It serves as a warming for potential misbehaviour

B. It promotes an orderly learning environment

C. It is easy to see who is listening or not

D. This gives time for other classroom tasks

216. Why should the chairs, tables and other physical features be arranged for flexible seating?

A. To facilitate interactive teaching-learning processes during class activities

B. To allow borrowing and sharing use of materials

C. To enable students to move around

D. To allow easy exchanges of seats as needed

217. As a classroom manager, teacher is generous with sincere praise. Which is an advantage
of this practice?

A. It conditions students’ discipline

B. It satisfies students’ need for praise

C. It is appreciated

D. It reinforces positive behavior

218. Should the students be involved in establishing routine for everyday tasks? Why?

A. Yes, they own the rules agreed upon

B. No, they will likely disobey it

C. Yes, for a maximum use of time

D. No, routine can change as needed


219. How can the teacher prevent an outbreak of untoward students’ behavior?

A. Put on a stern look as a warning to keep behaving

B. Show them she is aware of all activities happening in the classroom

C. Promise a prize if they behave well and do it

D. Punish misbehaviour with difficult activities

220. How should the teacher treat inattention and disruptions while the class is going on?

A. Send out the inattentive ones

B. Wait for awhile until they are through

C. Put on a stern look and gesture

D. She should not wait long, check early enough to avoid major problems

221. According to the principles on classroom management, teachers are bound by a sound
dictum which goes, “Prevention is better than cure”, what does this mean?

A. Do not wait for problems to erupt before you take move

B. Be very strict right from the start and do not postpone punishment

C. Disciplinary problems are in the hands of the teacher

D. Anticipate potential problems to come up anytime, and so, be ready

222. Teacher A is going to teach the parts of a human digestive system. Which best
instructional should be use?

A. Large drawing C. TV

B. Model D. Realia

223. In which level of the cognitive domain “demonstrate”, “compute” and “solve” are action
words that can be used in formulating objectives?

A. Comprehension C. Synthesis

B. Analysis D. Application

224. A teacher was almost annoyed when she saw a student run to the laboratory room to get
the materials that will be needed in the experiment. When asked why he did not wait for the
classmates to go there, he answered, “I like to be the one to do it.” What kind of motivation does
the student manifest?

A. Extrinsic C. External

B. Intrinsic D. Positive

225. When students communicate through body movements, such communication


is called___________________.

A. nondiscursive C. discriminating

B. perceptual D. reflex

226. A teacher asked, “If we raise the temperature to 100ºC what will happen?” Under which
kind of question does this fall?
A. Empirical C. Analytic

B. Evaluative D. Recall

227. What kind of questions should the teacher ask when she likes to elicit responses where the
student is free to generate independently his own idea? This question encourages originality,
flexibility and spontaneity.

A. Cognitive C. Divergent

B. Convergent D. Analytic

228. A student’s journal reveals significant information which may be used in a number of ways,
such as __________________.

I. determine their own learning

II. realize their own strength and weaknesses

III. realize what they enjoyed doing

IV. identify teacher’s mannerisms

A. I, II and III
B. I, III
C. II and IV
D. III and IV

229. Which famous narrations of scientists, inventors and historians can serve as sources of
significant information that can be used as part of a teaching methodology?

I. Anecdotes III. Autobiographies

II. Short stories IV. Diaries

A. I, II, III, IV C. I and II


B. I and III D. II and IV

230. It is essentially a way of acquiring knowledge and skills through direct and keen
observations followed by an analysis of what has been sensed and understood, thus results in
learning. This way of learning is known as ________________.

A. personal C. observational

B. experiential D. consequential

231. In a class of teen-agers a problem on how the authorities could solve the problem of
dealing with drug users and drug pushers was the lesson for the day. For a real-life search for
solutions, which teaching methodology is most appropriate?

A. Integrative approach C. Sociodrama

B. Debate D. Panel discussion

232. In any society, the key principles from the functionalist perspective include ____________.

I. Interdependency

II. Functions of social structure

III. Consensus and cooperations


IV. Equilibrium

A. I, II, III, IV C. I and II


B. I, III and IV D. III and IV

233. One of the for pillars of education is “ learning to know,” which includes listening, learning
to read with comprehension, observing and asking questions. What kind of skills should
students need to learn?

A. learn-to-learn C. evaluative

B. recall D. logical

234. Subcultures refer to cultural patterns that set apart some segments of a society’s
population. It can be based on _______________.

I. age III. Sexual preference

II. ethnicity IV. Occupation

A. I, II, III C. I and II


B. I, II, IV II and III

235. What is the major aim of multicultural education?

A. To create equal educational opportunities from diverse socioeconomic status, race, sex, age

B. To transform the school so that male and female experiences in a opportunity to learn

C. To help students acquire knowledge to function effectively in a democratic society

D. To help students develop a sense of personal and civic efficacy

236. The International Commission on Education for the 21st Century defines conscientization

A. the process of becoming aware of the contradictions existing within oneself and in society
and of gradually being able to bring about personal and social transformation

B. the dialectic process which starts with knowing oneself

C. the overall development of the individual as a human person and a member of society

D. the physical, intellectual and emotional integration of the individual into a complete man

237. A type of communication which refer to the use of gestures, facial expression and other
body movements is termed __________________.

A. auxiliary C. language

B. verbal D. non-verbal

238. Language is an abstract system of word meaning and symbol for all aspects of culture. It
includes _______________.

I. speech III. Numerals

II. written characters IV. Symbols

A. I, II, III, IV C. I, III only


B. I, II, IV D. III, IV only

239. Every society has a culture. Culture is defined as ___________________.


A. An organization of an phenomena, patterns of behaviors acquitted and transmitted

B. It is the learned norms that are communicated among the people

C. Sum total of symbols and ideas associated with a system

D. The set of learned behavior, beliefs, attitudes, values that are characteristic of a particular
society

240. In our culture, we hold beliefs about the characteristics associated with males and females.
As an example, males are rough, hyperactive and noisier, while females are gentle, soft-spoken
and obedient. This way of thinking is labelled as gender ________________.

A. stereotyping

B. schema

C. racism

D. inequality

241. Which important elements must the phase of curriculum Development consider according
to Ralph Tyler?

I. Learners III. Environment

II. Society IV. Subject matter

A. I, II and IV C. II, III and IV


B. I, III and IV D. I, II and III

242. Which basic philosophical system gave us the importance of mind and spirit and of
developing them in the learner and with whom is it closely identified ______________.

A. Idealism; Plato C. Pragmatism

B. Realism; Aristotle D. Perennialism; Adler

243. Upon which philosophy and whose philosophy is a curriculum whose fundamental idea is
described as “the world is a world of change; man can know anything within his experience;
hence the belief in learning by doing.”

A. Pragmatism; W. James

B. Idealism; Plato

C. Realism; Aristotle

D. Existentialism; Sartre

244. A major type of curriculum which puts emphasis on teaching facts and knowledge for future
use and has a well-defined scope and sequence is identified as __________________.

A. learner-centered

B. subject- centered

C. experience-centered

D. child-centered
245. In a 40-minute teaching period, a teacher in Science was observed to be asking “what”
questions and none of the “why” and “how” type. She seems to be focusing on ____________

A. teaching of facts

B. emphasizing understanding through a dynamic process

C. improving problem solving skills

D. the learners’ interest and needs

246. Which type of curriculum promotes a high level cooperative interaction on meaningful
immediate use of learning?

A. Learner-centered C. Society-centered

B. Subject-centered D. Culture-centered

247. A subject-centered which recognizes that students are not ready to learn certain concepts
until they reach the required level of development and maturity makes use of a specially
designed _______________ curriculum.

A. core C. correlated subjects-focused

B. spiral D. activity-based

248. Which type of curriculum stresses integration of learning by systemic correlation of subject
matter around themes drawn from the contemporary problems of living?

A. Core C. Correlated subjects

B. Fused subjects D. Broad-fields

249. In organizing and designing a curriculum, which criteria for the selection of content should
apply?

I. Validity III. Scope

II. Utility IV. Interest

A. I, II, III, IV C. III and IV only


B. I and II only D. Interest

250. Teacher A prefers the use of experiential rather than rote learning, hence focuses on
“custom-made” activities for children and a generous use of motivation. This curriculum design
is classified as ______________.

A. Child-centered C. subject centered

B. process-centered D. activity-based

251. Peter kisses the hand of his mother before leaving and upon arrival from a long trip. His
youngest son learned to do same when he left for US to study. This is an example __________.

A. acculturation C. tradition

B. enculturation D. value formation

252. The mayor of the town recited in front of the townsfolk, “My right ends where the right of
other begins”. This is interpreted as __________________.

A. rights are inalienable C. rights are alienable


B. rights are absolute D. rights are not absolute

253. When a teenager was given a scholarship to study in Malaysia, he was surprised at the
reactions, traditions and cultural practices of his classmates’ families. Understandably he
suffered from a/ an _________________.

A. culture relativity C. indoctrination

B. culture shock D. exemplification

254. Which is the most authentic method of assessment?

A. Performance-based C. Pencil-and-paper

B. Objective D. Traditional

255. Teacher M taught her Science students the parts of the digestive system. Then she gave a
10-item test to find out the parts of the system which they failed to learn. What kind of
assessment was done?

A. Formative C. Summative

B. Diagnostic D. Placement

256. It is a common practice to give a pre-board examination. If its results can explain how the
students will likely perform in their licensure examination, what type of validity does the pre-
board exam possess?

A. Predictive C. Construct

B. Concurrent D. Concluding

257. For purposes of evaluation, Teacher A required a collection of the student’s best works, in
the form of products and accomplishments for a certain period. This type of
assessment refers to __________________.

A. diary C. observation report

B. portfolio D. tests result

258. Which is a measuring instrument that offers a set guidelines or descriptions in scoring
different levels of performance or qualities of products of learning.

A. Products-based C. Rubric

B. Performance-based D. Portfolio

259. A teacher wants to make a scoring rubric for a student’s output. Which format will use
differential weights for the qualities of a product or a performance?

A. Analytic rubric C. Rating scale

B. Holistic rubric D. Performance- based

260. The grade six pupils were given a diagnostic test on determining criteria for classifying
vertebrates to find out if they can proceed to the next phylum. The results of the test were very
low. What should the teacher do?

A. Count frequency of errors to find out the lessons that the majority of students need to
relearn

B. Proceed to the next lessons to be able to accomplish more


C. Construct a parallel test to find out if the same results will be obtained

D. Record the scores then inform the parents of the poor performance of each pupil

261. If the result of a test showed a negatively skewed distribution, this is an indication that test
is _____________________.

A. very easy C. average

B. very difficult D. moderately difficult

262. Which field of study aims to create equal educational opportunities for students from
diverse racial, ethnic, social class and cultural groups.

A. Liberal education C. Cultural socialization

B. Multicultural education D. moderately difficult

263. When males and females have equal opportunities to realize their own full human rights
and contribute to and benefit from economic, social and cultural development, we
have attained __________________.

A. gender equality C. socialization process

B. patriarchy D. equality of access

264. One of the trends and developments in Philippine Education is the trifocalization of the
educational system. The government established _______________________.

A. Basic Education and Higher Education

B. DepEd, TESDA and CHED

C. Elementary and Secondary Education

D. Alternative Education

265. The 2008 education reform measures recommended by the President’s Task Force for
Education ( PTFE ) are found in the book ___________________.

A. Our Common Future in Philippine Education

B. Four Pillars of Education

C. The Education of Young People and Adults in the Philippines

D. Philippine Main Education Highway: Towards a knowledge-Based Economy

266. Which is one of the key features of 21st century Education?

A. Knowledge as the Only Learning Outcome

B. Traditional Pedagogies

C. Rigid Subject Matter Boundaries

D. Lifelong Education for all

267. The UNESCO J Delors Report identified four pillars of learning:

I. Learning to live together

II. Learning to be
III. Learning to do

IV. Learning to know

A. I C. IV
B. II D. III

268. The basic value that is central to Human Rights education is __________________.

A. justice for all C. human solidarity

B. human dignity D. respect for diversity

269. When a public official is a member of a board or a stockholder of a private corporation, and
the interest of such corporation or his rights and duties therein may be opposed to the faithful
performance of his official duty, such a conduct results in what is considered ______________.

A. divestment of interest C. professionalism

B. conflict of interest D. discriminatory

270. The Revised Policies and Standards for Undergraduate Teacher Education Curriculum
which is the current requirements before one is qualified to take the licensure
examination is _____________________.

A. CMO No. 30,s 2004 C. CMO NO. 20,s 2005

B. CMO No. 31,s 2005 D. CMO No. 21,s 2004

271. Which of the following can result in the revocation of a teacher’s license or suspension
from the practice of the teaching profession?

I. Conviction of a criminal offense

II. Immoral, dishonorable conduct

III. Malpractice, gross incompetence

IV. Habitual use of drugs

A. I, II, III, IV C. II and IV only


B. I and II only D. III and IV only

272. Upon passing this Licensure Examination for Teachers ( LET ) what will be issued to you?

A. Certificate of Passing the Licensure Examination

B. Certificate of Registration and Professional License

C. License to Practice Teaching signed by the Broad

D. Certificate of Professional Registration

273. Academic freedom shall be enjoyed by teachers in the discharge of their professional
duties, with regard to teaching and classroom methods. Such a right is provided in the
_______________.

A. Magna Carta for Public School Teachers

B. Education Act Establishing an Integrated System

C. Philippine Constitution
D. Professionalizing Teaching

274. Physics concepts were learned through a 40-minute TV broadcast.The principal required
his 3 nonmajors to view the lessons at home every Saturday and after 3 to 5 episodes took a
test prepared by an expert to find out if they learned the concepts. Such a practice can be
considered _________________.

A. distance learning C. use of media

B. self-learning D. informal learning

275. Choose the guiding principles in the selection and utilization of educational technologies.

I. Meaningfulness III. Self-paced

II. Purpose IV. Communication effectiveness

A. I, II, IV C. II, III, IV


B. I, II, III D. I, III, IV

276. What is the battle cry of President Arroyo’s moral renewal program?

A. Bayan muna bago sarili

B. Zero tolerance for corruption

C. Honesty in public service

D. Responsible citizenship

277. Concientization as pedagogy and an approach to teaching is effective in promoting


____________________.

A. critical awareness of issues

B. ecological awareness

C. political activism

D. lack of trust in political leaders

278. If we aim to produce globally competitive graduates, the Philippine Education should give
major emphasis on ___________________.

A. Technology and Citizenship Education

B. English, Science and Mathematics

C. Bilingual Education and Values Education

D. Humanities and Work Education

279. Which of the following is the first target of the Millennium Development Goals ( MDG )
formulated by members states of the UN in September 2000?

A. Achieve universal access to primary education

B. Reduce child mortality

C. Ensure environmental sustainability

D. Eradicate extreme poverty and hunger


280. Alternative education is a basic right and will be free for all Filipinos. What are the
characteristics of alternative education?

I. Nationalist III. Pro-people

II. Scientific IV. Democratic

A. I, II, III, IV C. II and III only


B. I and IV only D. I and III only

281. Section 13 of RA 4670 states that teachers are required to render no more than
_____________ hours of actual classroom teaching a day.

A. 6 C. 7

B. 5 D. 8

282. Which law is intended to strengthen Teacher Education in the Philippines by creating
Centers of Excellence ( COE )

A. R.A 7784 C. R.A 7896

B. R.A 7836 D. R.A 7722

283. The term “Board” in the Teachers Professionalization Act of 1994 refers to ____________.

A. Board of Commissioners

B. Board of Professional Teachers

C. Board of Licensure Examinations

D. Professional Commissioners

284. A computer is used to present instructions directly to students, called computer-assisted


instruction ( CAI ). What are advantages of CAI?

I. Can model a real system or situation

II. Can act as a tutorial

III. Increases motivation effect

IV. Encourages student participation

A. I, II, III, IV C. I and III


B. I and II D. I and IV

285. School A is going to buy a computer. Aside from it being a communicative, informative and
a constructive tool, what other role should the school consider?

A. Situating C. Utility

B. Application D. Self-instructional

286. Teacher R and student meet at the same time but in different places as in an audio
teleconferencing. This type of distance education program is called ______________.

A. synchronous C. electric boards

B. asynchronous D. hybrid courses


287. What are the three major reasons why internet can be effectively used by teachers and
students. These categories of use are communication as in interpersonal exchanges, general
sources as in access to a museum collection and the third category is ______________.

A. information retrieval as in tutorial sites

B. information exchanges as in favourite quotes

C. information collection as in book reviews

D. instant messaging

288. As a communicative tool of computers, the email or electronic bulletin boards are classified
as ________________. Why?

A. asynchronous; exchange of information is not “live”

B. synchronous; simultaneous communication

C. productivity tool, information collection as in book reviews

D. productivity tool, manipulating information

289. The advantages in using educational technologies like audio visuals, motion pictures and
media like TV and films are: a) can show motion, b) allows real-life, and c) _______________.

A. complex treatment of an issue

B. sometimes can be static

C. programs run at a fixed pace

D. allows repeated viewing of performance

290. Which is true of present Basic Education Curriculum? It is for the __________________.

A. development of an empowered learner who possesses life skills for lifelong learning

B. promotion of nationalist and dedicated Filipino citizens

C. promotion of a just and humane society

D. development of a highly competitive Filipinos for globalization

291. In what social indicator is the Philippines in the Top 10 in the world’s ranking?

A. Gender equality C. Good governance

B. Food security D. Respect for human right

292. Republic Act 7877 signed into law on February 14, 1995 declares as unlawful
_________________.

A. sexual harassment in the workplace

B. hazing in fraternities

C. tuition fee increase in basic education

D. cutting of trees in the highways

293. Using history books featuring defeats and weaknesses of the Filipino as a people, what
should a teacher do?
A. Relate facts that may enlighten and inspire them

B. Present them and deny the causes

C. Do not highlight such events in the discussion

D. Point their irrelevance to the lesson

294. Education as the foundation of Sustainable Development was reaffirmed at the


_____________.

A. Johannesburg Summit, 1992

B. Millenium Goal Summit, 1995

C. World Summit on Educational Development, 2002

D. Action on Education For All, 2000

295.Which of the following can be considered a form of civic engagement?

A. Electoral participation C. Critical of g

296. The provision of academic freedom is applicable for teachers in ________________?

A. colleges and universities C. basic education teachers

B. private tertiary institutions D. public schools only

297. Which are the basic dimensions of the teaching profession?

A. Knowledge, Social, Ethical and Professional

B. Knowledge, Economic, Moral and Social

C. Professional, Social, Emotional and Economic

D. Security, Professional, Social and Knowledge

298. Article XIV, 1987 Philippine Constitution, stipulates among others _______________.

A. highest budgetary allocation for education

B. promotion of science, math and technology

C. private ownership of educational institution

D. promotion of physical education and other programs

299. Education Act 1982, declares among others, the _____________________.

A. rights, duties and responsibilities of parents and students

B. obligations of non-teaching personnel

C. special rights of academic staff

D. rights of school administration

300. Which refers to the education of persons who are physically, mentally, and socially
different from the “normal” individuals.
A. Alternative Learning System C. Normal Education

B. Special Education D. Early Childhood Education


301. Which refers to organized school- based activities aimed at attaining specific learning
objectives for the illiterates and out-of-school youth and adults?

A. Special Education C. Practical Arts Education

B. Alternative Learning System D. Informal Education

302. Section 24 of RA 4670 provides that teachers are entitled to a study leave not exceeding
one year after how many years of service?

A. 7 C. 10

B. 8 D. 5

303. Who/what should be consulted in the formulation of national educational policies


standards, and in the formulation of national policies governing the social security of the
teachers?

A. National Teachers Organization

B. Secretary of Education

C. Government Service Insurance System

D. Department of Education

304. No person shall practice the teaching profession in the Philippines without having
previously obtained a ______________________.

A. valid certificate of registration and a valid license

B. certificate of having passed the licensure exam

C. signed identification card issued by PRC


D. valid document of examination passed

305. The regular progression of Filipino teacher’s salary from minimum to maximum is done
through ___________________.

A. a regular increment every 3 years

B. an increment after 5 years

C. an increment after 10 years

D. a regular increment every year

306. A teacher who is a BSE graduate, has not passed the LET yet. Can she be hired?

A. Yes, on a provisional status for not less than 1 year


B. Yes, on a permanent status
C. No, unless for emergency D. Not at all

A. BIOLOGY AND ENVIRONMENTAL SCIENCE

1. Which blood type has neither anti-A nor anti-B and can receive blood from anyone?

A. AB C. B

C. A D. O
2. The kidneys contain millions of excretory units or filtering units which is made-up of a network
of glameroli. These units are called _____________________.

A. lymph nodes C. renal tributes

B. nephrons D. ureter

3. Identify the steroid hormones among the following:

I. androgens

II. estrogens

III. oxytocin

IV. progesterone

A. II, III and IV C. I, II, and IV


B. II and III D. I and III

4. Which is the technology being used now in the surgery of the retina?

A. MRI C. CT scan

B. X-ray D. Laser

5. Which part of the brain controls heart action, breathing and digestion?

A. Medulla oblongata C. Cerebrum

B. Cerebellum D. Thalamus

6. When the air sacs of a smoker’s lungs become brittle, the patient experiences
breathlessness. It is a condition called ___________________.

A. pneumonic C. pleurisy

C. emphysema D. lung cancer

7. Which of the following techniques will enable us to detect the defects of the fetus as early as
the 8th and 12th week of pregnancy?

A. Amniocentesis C. Bar body test

B. Ultrasound D. Chorionic sampling

8. Using a genetic engineering technique, a research team was to produce a rare human protein
which is now being tired as a possible cure for cancer. Identify this protein.

A. Keratin C. Interferon

B. Collagen D. Polymerase

9. Which parental pair can produce a colorblind female?

A. Colorblind mother and normal vision father

B. Heterozygous normal vision mother and colorblind father

C. Homozygous normal vision mother and colorblind father

D. Heterozygous normal vision mother and normal vision father


10. What is the process that uses microbes to break down pollutants in the environment,
particularly the soil?

A. Biolimatnion C. Bioremediation

B. Pollutant breakdown D. Environment eradication

11. Who among the following scientists supported the idea that evolutionary changes occurred
because there was need for them?

A. Darwin C. De Vries

B. Lamarck D. Mendel

12. Which is the connective tissue that is made up of tough fibers that are strong enough to
connect a bone on our body?

A. Tendon C. Joints

B. Cartilage D. Ligament

13. Name the kind of organelle that is found in the cytoplasm which is composed of protein and
lipid molecules and is considered the powerhouse of the cell?

A. Mitochondria C. Centrosome

B. Lysosome D. Golgi bodies

14. Fishermen are advised not to drink ocean water because it can cause _________________.

A. dehydration of the cells C. increase in cell pressure

B. busting of the cells D. poisoning of the cells

15. Name the feeding procedure unique to birds wherein the mother bird partially digests the
food and places an amount into the young’s beak?

A. Redigestion C. Regurgitation

B. External digestion D. Internal digestion

16. Describe the chromatopores that lizards and snakes possess

A. Green-colored cells used for protection

B. Pigment cells that enable them to change color

C. Colored pigments used for defense

D. Colored pigments used food-getting

17. The “caste system” exhibited by honeybees wherein each member has a specific task to
perform is called ____________________.

A. polymorphism C. morphologic

B. metamorphosis D. adaptive mechanism

18. How do double organic molecules travel from photosynthesis sites or storage areas to the
organs that need them? What do we call this “long distance transport” of solutes?

A. Translocation C. Transduction
B. Diffusion D. Transpiration

19. Name the plant hormone that promotes stem elongation and helps break dormancy of seeds
and buds.

A. Florigen C. Gibberellins

B. Abscisic acid D. Cytokinins

20. Which part of the flower of the cashew plant develops into a fleshy part where the seed is
attached?

A. Ovary C. Pistil

B. Receptacle D. Stigma

21. Bile which is used to emulsify fat is stored in the ________________.

A. liver C. gall bladder

B. urinary bladder D. kidney

22. Which is the emergency hormone?

A. Throxine C. Ptyalin

B. Pitvitrin D. Adrenalin

23. The most important on-going project today which involves the mapping of the genes in all
our 46 chromosomes is the __________________.

A. Human Genome project

B. Stem Cell production

C. Cloning of organisms

D. Genetic Engineering Project

24. I a double-stranded DNA, which of the following are complimentary to each other?

A. Members of a base pair C. Thymine and cytosine

B. Ribose and organic base D. Adenine and guanine

25. In the creation of a genetically modified organism in the country today, what microbial
organism is being used?

A. Pseudomonas vulgaris C. Bacillus thuringensis

B. Agrobacterium D. Escherichia coli

26. Which stage in the life cycle of a butterfly is considered most destructive by farmers and
why?

A. Eggs; it spoils the taste of leaves

B. Larva; it eats the parts of the plant

C. Adult; it kills the flowers

D. Pupa; it adds to the weight of the branch


27. Why do leaves of plants wilt temporarily at noontime?

A. Transpiration occurs faster than root absorption of water

B. Roots absorb water that the leaves evaporate

C. Too much heat makes the leaves weak

D. Osmosis of the water through the stem is faster

28. Which of the following is a possible food chain in a particular area?

A. cat corn rat

B. corn rat cat

C. rat corn cat

D. corn cat rat

29. In an energy pyramid which group of organisms receive the least amount of energy?

A. Producers C. Second-order consumer

B. First-order consumer D. Third-order consumer

30. In what way do we become part of the oxygen-carbon dioxide cycle?

A. We use oxygen and carbon dioxide to keep us alive

B. We give off oxygen during respiration

C. We breath oxygen and give off carbon dioxide

D. We take in carbon dioxide and use oxygen for respiration

31. What kind of biome is prevalent in the equatorial zone of the earth, to which our country
belongs?

A. Taiga C. Tropical rainforest

B. Tundra D. Freshwater

32. How will you determine the population density of grass growing in the school lawn?

A. Measure the are and count the number of grass

B. Measure the area occupied by one grass

C. Count the big grass in the area during one month

D. Count the grass growing in one year

33. In a particular ecosystem, what trophic level is occupied by the herbivores?

A. Primary consumer C. Consumers

B. Producers D. Decomposers

34. What interspecific relationship may exist between frogs and fishes in a pond ecosystem?

A. Parasitism C. Competition

B. Predation D. Commercialism
35. Which of the following factors contribute to an increase in a town’s human population?

A. Immigration and natality

B. Emigration and mortality

C. Immigration and mortality

D. Emigration and natality

36. Which is the property of a particular species living in a particular setting together with the
role it plays in relation with other species?

A. Habitat C. Population

B. Niche D. Community

37. Choose from among these heterotrophs organisms that feed on partially decomposed
tissues of plants and animals.

A. Herbivores C. Detritivores

B. Autotrophs D. Carnivores

38. Which is the biogas obtained from biological wastes such as those from animal manures
which contains hydrocarbons?

A. Methane C. Ester

B. Alcogas D. Ethanol

39. One of the main features of man-made aerosol is a propellant which includes such gases as
butane and chlorinated hydrocarnons. Why is this group seen as a dangerous pollutant?

A. Flourocarbons break down and release carbon monoxide

B. Contains insecticides that can eliminate pests

C. Could act as pollutants of bodies of water

D. Flourocarbons does not cause thermal pollution

40. What is a disposal area where solid wastes are deposited, spread in layers, compacted and
covered with soil daily?

A. Open Dump C. Sanitary landfill

C. Incinerator D. Recycling dump

B. PHYSICAL SICIENCES, EARTH SCIENCE FOR BEEd

1. The average salinity of the ocean is ________________?

A. 25% C. 15%
B. 35% D. 45%

2. In which layer of the atmosphere does weather occur?

A. Stratosphere C. Troposphere

B. Ionosphere D. Mesosphere

3. What motion of the earth causes the occurrence of night and day?

A. Revolution C. Solar translation

B. Rotation D. Inclination of axis

4. When does a summer solstice occur and describe it

A. June 21 and 22 wherein days are longer than night

B. March 21 and 22, there is equal length of day and night

C. September 22 or 23, nights are longer than days

D. December 21 and 22, nights are longer than days

5. What instrument is used to measure wind velocity?

A. Clinometer C. Anemometer

B. Thermometer D. Hygrometer

6. Identify our galaxy and to which type does it belong?

A. Andromeda; spiral C. Pegasus; irregular

B. Milky Way; spiral D. Magellanic; irregular

7. Which surface on earth heats and cools faster?

A. Atmosphere C. Water

B. Vegetation D. Soil

8. What forces keep the planets in their orbits?

A. Magnetism and inertia C. Revolutionary forces

B. Inertia and gravity D. Gravity and electromagnetism

9. A lunar eclipse can occur during ___________________.

A. new moon C. last quarter

B. first quarter D. full moon

10. Jet propulsion is explained by the Law of __________________.

A. Acceleration C. Interaction

B. Inertia D. Universal Gravitation

11. Why can the astronauts jump higher on the moon than on earth?

A. Moon’s gravity is one-sixth that of earth


B. There is no atmosphere on the moon

C. Their mass is less

D. Moons gravity is more than one

12. Mirrors can form images due to the ________________.

A. regular refraction of light

B. difference in density of light

C. regular reflection of light

D. separation of white light

13. In the electromagnetic spectrum, how will the waves be arranged in the order of increasing
wavelength?

A. Gamma, ultraviolet, infrared, radio

B. Radio, gamma, ultraviolet, infrared


C. Radio, infrared, gamma, ultraviolet

D. Gamma, radio, infrared, ultraviolet

14. Light falling on soap bubbles form a band of colors. This is due to _________________.

A. dispersion C. refraction

B. interference D. diffraction

15. The voltage across the input terminals of a transformer is 110V. The primary has 50 loops
and the secondary has 35 loops. What is the voltage that the transformer?

A. 55 V C. 110 V
B. 65 V D. 220 V

16. According to Ohm’s law, the amount of current in a circuit is directly proportional to the
voltage and ____________________.

A. directly proportional to the resistance

B. inversely proportional to the resistance

C. inversely proportional to the voltage

D. directly proportional to the voltage

17. Which part of the nucleus of an atom determines its isotope?

A. Proton C. Positron

B. Electron D. Neutron

18. The sun emits a tremendous amount of energy which is the result of atomic
______________.

A. fission C. fusion

B. disintegration D. intensity
19. The half-life of a certain isotope is one day. At the end of 2 days, how much of the isotope
remains?

A. One half C. One third

B. One eight D. One quarter

20. Hydrogen peroxide, H2O2 can be brought as a disinfectant for wounds. It is composed of
6.00% hydrogen and 94.00% oxygen. This illustrates the Law of _______________.

A. Conservation of Mass C. Conservation of Matter

B. Definite Proportion D. Multiple Proportion

21. Why do farmers practice spreading powered limestone on their fields?

A. It acts as fertilizer

B. It improves the absorbing ability of the soil

C. It neutralizes soil acidity

D. It provides the needed nutrients

22. Who is the French physicist who, while studying about fluorescence working with uranium
compound, accidentally discovered radioactivity?

A. Marie Curie C. William Roentgen

B. Antoine Becquerel D. John Dalton

23. Which kind of radiation is most dangerous to humans?

A. Gamma C. Alpha

B. Beta D. Infrared

24. Why is carbon-dating the best way to determine the age of fossils?

A. Plants and animals emit Carbon-14

B. After death, a plant or animal stops producing oxygen

C. After a plant or animal dies it stops taking in Carbon-14

D. After death, a plant or animal stops producing carbon dioxide

25. The intensity of cosmic rays bombarding the earth’s surface is largest at the ____________.

A. Mid-latitudes C. near the equator


B. Equator D. poles

26. A volcanic cone described to have a steep slope with a narrow base is called
_______________.

A. shield C. composite

B. cinder D. lava

27. Which is a measure of the strength of an earthquake?


A. Magnitude C. Intensity

B. Tremors D. Focus

28. The core of the earth is composed of __________________.

A. iron and sodium C. nickel and magnesium

B. iron and nickel D. magnesium and iron

29. Climate in a region is determined by ___________________.

A. temperature and rainfall

B. wind velocity and rainfall

C. cloudiness and temperature

D. latitude and temperature

30. During the earth’s revolution seasons occur due to the __________________.

A. inclination of the earths orbit

B. inclination of the earths axis

C. rate of earth’s revolution

D. rate of earth’s rotation

31. Name the seasonal wind that is experienced from December to February, also called
“hanging amihan.”

A. Southwest monsoon C. Northeast monsoon

B. Tropical winds D. Trade winds

32. Choose the color of the stars with the highest temperature

A. Red C. Orange

B. Yellow D. Blue-white

33. The most widely accepted theory on the origin of the universe is the ________________.

A. Steady State C. Big Bang

B. Pulsating D. Continental Drift

34. Which is the first mission and when did it land on the moon?

A. Apollo 13; July 16, 1969

B. Apollo 12; July 19, 1969

C. Apollo 11; July 21, 1969

D. Apollo 14; July 20, 1969

35. Why do we see the sun rising in the east?

A. Earth rotates counterclockwise

B. Earth revolves from east to west


C. Earth rotates clockwise

D. Earth’s axis is inclined

36. The breaking of boulders into small particles due to the action of wind and water is called
___________________?

A. erosion C. deflation

B. weathering D. faulting

37. What is the process that occurs when exposed rock surfaces break due to daily changes in
temperature?

A. Exfoliation C. Hydration

B.. Deflation D. Dissolution

38. Name the processes involved in the occurrence of the hydrologic cycle.

I. Evaporation

II. Condensation

III. Air circulation

IV. Precipitation

A. I, II and IV C. II, III and IV

B. I, II and III D. I, III and IV

39. How long does earth complete one journey around the sun?

A. 364 days C. 24 hours

B. 365 ¼ days D. 27 1/3 days

40. We see the four phrases of the moon in how many days?

A. 27 1/3 days C. 29 ½ days

B. 31 days D. 30 days

41. Which of these would be the best conductor of electricity?

A. Saltwater C. Glass

B. Space D. Air

42. What does the needle of a compass respond to when it deflects?

A. Earth’s geographic pattern C. Earth’s magnetic field

B. Earth’s gravitation D. Earth’s electric field

43. Which among the simple machines work to change the direction of the force?

A. Lever C. Wheel and axle

B. Pulley D. Wedge

44. Two charges separated by a distance of 1 meter exert a 1-N force on each other. If the
charges are pushed to separate at ¼ m, what will be the force on each charge?
A. 8 N C. 1 N

B. 16 N D. 4 N

45. If you comb your hair and the comb becomes positively charged, what happens to your
hair?

A. hair becomes positively charged

B. Hair becomes uncharged

C. Hair becomes negatively charged

D. Nothing happens to the hair

46. Which among the following illustrates the occurrence of lighting?

A. Charging by polarization C. Charging by friction


B. Charging by induction D. Charging by contact

47. Which can help prevent fire by blocking the path of electricity?

A. Switch C. Resistor

B. Capacitor D. Fuse

48. Choose which material could be used as insulator

A. Iron C. Plastic

B. Copper D. Aluminum

49. If a bar magnet is suspended by a string from its center, what will happen to it?

A. It will line up with the earth’s magnetic field

B. It will line up with the earth’s gravitational field

C. It will keep swinging

D. It will line up in any direction

50. Which of the following is a characteristic of a series connection?

A. The sun of the resistance is less than the total

B. There is only one current through each load

C. There is a common voltage drop across all load

D. The current flowing across each load varies

ENGLISH for BEEd

1. Listening is defined as constructing meaning by:

I. Predicting
II. Making connections

III. Focusing

IV. Expressing

A. I, II and III C. I, II and IV


B. II, III and IV D. I, III and IV

2. Describe open-ended or divergent questions

A. Broad and may have multiple answers

B. Elicit specific information

C. Elicit prior knowledge

D. Asks for ones experience

3. Idiom is a phrase whose usual meaning do not make sense. Choose the idioms in the
following phrase.

A. Burn disposal wastes

B. Burn the midnight oil

C. Bark during the night watch

D. Bark when disturbed

4. Which is an alphabetically arranged list of significant words or terms used in the resource
material?

A. Appendix C. Glossary

B. Preface D. Index

5. A student is searching for the synonyms of a word. Which should she consult?

A. Encyclopedia C. Dictionary

B. Thesaurus D. Atlas

6. What reading resource contains information about the theories on the origin of the solar
system?

A. Atlas C. Almanac

B. Yearbook D. Dictionary

7. Which graphic organizer will you use to show the similarities and differences between two
things by using overlapping shapes?

A. Concept map C. Fishbone diagram

B. Venn diagram D. Tree diagram

8. The stranger who climb the fence is CULPUBLE. This means he is _________________.

A. innocent C. guilty

B. strong D. an intruder
9. To illustrate the four stages of child development according to Piaget, which organizer will
you use?

A. Venn Diagram C. Web

B. Flow chart D. T-chart

10. Which is the instructional procedure used when teachers gather information about reader’s
abilities to deal with the content and structure of texts read?

A. Cloze procedure C. K-W-L chart

B. Cubing D. Grand convention

11. Children who have been exposed too much to TV usually suffer from short
______________.

A. attention span C. understanding

B. concentration D. memory

12. Life, so they say is just a game

And they let it slip away

Don’t let life slip away

Grasp it while you can

We may never pass this way again

What does the stanza suggest?

A. Opportunity comes once, grab it


B. There is a second life anyway
C. Life will be gone in a second
D. We will all pass away

13. A low pressure area was sighred 120 km from Basco, Batanes. A depression is expected to
occur in Region I. This is an example of a reading skill called __________________.

A. predicting C. drawing conclusion

B. perceiving D. inferring

14. The world lackadaisical was used to describe the student’s indifference. They are called
___________ .

A. antonyms C. synonyms

B. explanations D. clues

15. What does the expression “Love begets love” mean?

A. Loving is a one-way agreement between two persons.

B. You love others but you may not get love in return.

C. If you love other people do not expect love always.

D. If you exude with love, you will reap love in return.

16. News travels as fast as the wind. This line is a ________________ .


A. simile C. hyperbole

B. metaphor D. personification

17. To apply for a passport some important papers must be presented. Which of the following
statements refers to papers as documents?

A. They passed the papers after filling the blanks.

B. Jane reads the paper early in the morning.

C. The registrar evaluated the papers I presented.

D. Always put the waste papers in the garbage can.

18. To develop readin g skills which instructional materials will you use?

I. Flashcards II. Puppets III. Television IV. Radio

A. I,II and III C. I, II and IV

B. II, III and IV D. I, II, III and IV

19. Supporting materials in the form of tabulations, charts and diagrams about the content being
presented usually appear in the ___________ .

A. glossary C. index

B. body D. appendix

20. Formulating a statement based on an observation but will still need further evidences
contribute to the development of one’s reading skill. Name this skill.

A. Concluding C. Generalizing

B. Inferring D. Analyzing

21. It consists of alphabetically arranged words, together with their meanings, syllabications and
pronunciations and other pertinent information. Identify this reference resource.

A. Encyclopedia C. Thesaurus

B. Dictionary D. Yearbook

22. It is a list of titles discussed in the content, arranged alphabetically, together with the page
location which helps the readers immensely. It is the ______________ .

A. index C. table of contents

B. appendix D. glossary

23. Which is a very common reading resource used daily with important news, advertisements
and editorials?

A. Magazine C. Newspaper

B. Journal D. Newsletter

24. Deforestation has been blamed by lowland settlers as the main cause of flooding which
originate from the nearby hills. Reading skills can be enhanced when a student can perceive a
relationship such as _____________ .

A. opinion of townfolks C. forming an inference


B. actual observation D. a cause and effect

25. It may seem FRIVOLOUS to you, but it’s important to me. Frivilous means ____________ .

A. worthless B. vital

B. significant D. practical

FILIPINO for BEED

1. Alin sa mga sumusunod na pangungusap ang may paksa?

A. Kay ganda ng paglubog ng araw.


B. Nagbabasa sila ng sa aklatan.

C. May pasok ba bukas?

D. Mainit ngayon.

2. Tukuyin ang uri ng pangungusap na itong walang paksa. “Walang anuman.”

A. Pormulasyong panlipunan C. Eksistensyal

B. Patanong D. Panagot

3. Alin ang paksa na sumusunod na pangungusap?

“Ginagawa niya ang pagdarasal araw-araw.”

A. Ginagawa C. Pagdarasal
B. Araw-araw D. Niya

4. Ang panalangin ay mabisang sandata sa buhay. Ito ay isang ____________ .

A. pagwawangis C. onomatopia

B. pagtutulad D. personipikasyon

5. Alin sa mga sumusunod ang tinatawag na nominal?

A. Pananda at pang-ukol C. Pangngalan at panghalip

B. Pang-abay at pang-uri D. Pantukoy at pangatnig

6. “You can count on me.” Ang pinakamalapit na salin nito ay ___________ .

A. ibilang mo ako C. maaasahan mo ako

B. bilangin mo kami D. maasahan mo sila

7. Ang katawagan sa mga kataga o tunog na isinasama sa kayarian ng mga pangungusap


katulad n grin, din, ba, lang, pa at na ay mga ______________ .

A. digrapo C. ponema

B. ingklitik D. morpema

8. Ano ang katawagang ibinibigay sa mga bahagi ng pananalita na pang-ukol, pangatnig at


pang-angkop?

A. Pangdamdamin C. Pangkayarian

B. Pangnilalaman D. Pang-asimilasyon

9. Ito ay pag-aaral ng mga ponema ng isang wika. Ito ay _____________ .

A. leksikon C. ponolohiya

B. monopolohiya D. sintaks

10. Ito ay tulang pasalaysay na kinapapalooban ng mga pakikipagsapalaran, pinagmulan at


buhay ng isang tauhang may pambihirang katangian.

A. Epiko C. Awit

B. Alamat D. Bugtong
11. Sino ang unang bumuo ng titik n gating pambansang awit na may pamagat na Himno
Nacional Filipino ng panahon ng himagsikan?

A. Jose Palma C. Marcelo H. del Pilar

B. Andres Bonifacio D. Graciano Lopez Jaena

12. Ang magasing ito ay unang nilimbag noong 1922 na kababasahan ng mga nobela, maikling
katha at iba pang pangkaalamang lathalain.

A. Bulaklak C. Ilang-ilang

B. Liwayway D. Hiwaga

13. Alin ditto ang mga salita o lipon ng mga salita na ang kahulugan ay iba sa kahulugan ng
mga salitang binubuo?

A. Pahayag C. Ekspresyong idyomatiko

B. Salitang upemistiko D. Tayutay

14. Ano ang kayarian ng pangungusap na ito? “Nakipagkita sa Pangulo ang mga senador at
kinatawan sa Batasan pambansa.”

A. Langkapan C. Payak

B. Tambalan D. Hugnayan

15. Ano ang aspeto ng pandiwa sa pangungusap na ito? “Mag-antay ng tahimik sa salas.”

A. Pawatas C. Imperpektibo

B. Kontemplatibo D. Perpektibo

16. Damhin mo ang init ng pagmamahal ng iyong magulang. Ang salitang nasa malaking titik ay
nasa anyong pandiwang di karaniwan na _____________ .

A. may kutad C. may palit

B. may pungos D. may kaltas

17. Ibigay ang pokus ng pangungusap na ito. “Ikinaligaya ko ang pagdating mo.”

A. sanhi C. actor

B. lokatibo D. layon

18. Magsasaliksik ang mga mag-aaral sa internet ng kanilang takdang aralin. Ibigay ang aspeto
ng pandiwa.

A. Perspektibo C. Imperpektibo

B. Kontemplatibo D. Pawatas

19. Alin ditto ang pagsusulit na madaling ihanda ngunit mahirap iwasto at ang guro lamang ang
dapat magwasto?

A. Pagkilala C. Pasanaysay

B. Punan ang patlang D. Tukuyan

20. Ito ang gamit ng guro na pinagkukunan ng mga kompitensi na dapat matupad sa pagtuturo.
A. Lubusang pagkatuto C. Kompetensi ng Aralin

B. Kompitensi sa pagkatuto D. Takdang aralin

21. Ito ay kalipunan ng mga araling nakaayos ayon sa layunin at mga mungkahing paraan kung
paano ito itututro.

A. Pamphlet C. Manwal ng guro

B. Teksbuk D. Pantulong na aklat

22. Ang plano kung ano ang ituturo at paano ito itututro ay napapaloob sa ____________ .

A. silabus C. portfolio

B. aklat D. kagamitan

23. Alin ang pinakamalapit na salin sa sumusunod na pangungusap, “ I will praise my God all
the days of my life.”

A. Pupurihin ko ang Diyos sa araw-araw.

B. Ako ay magpupuri sa Diyos.

C. Pupurihin ko ang Diyos sa buong buhay ko.

D. Purihin ang Diyos sa araw-araw.

24. Sa pangungusap na “Panahon na upang MAGDILAT NG MATA” at makisangkot sa mga


usapin. Ano ang ibig ipahiwatig nito?

A. Umiwas sa usapin C. Idilat ang mata

B. Magising sa katotohanan D. Kalimutan ang isyu

25. Ito ang kuro-kuro ng patnugot hinggil sa mahalaga at napapanahong isyu ng araw na iyon.

A. Lathalain C. Editoryal

B. Balita D. Pitak

26. Ibigay ang kayarian ng pangungusap na ito. “Kung aalis k asana ay iwan mo ang dadalhin
ko sapagkat umaaraw ngayon kaya ako ay tutuloy sa Mega Mall.”

A. Payak C. Hugnayan

B. Langkapan D. Tambalan

27. Ang teoryang ito ay nakatuon sa bias ng panitikan sa kaasalan, kaisipan at damdamin ng
tao. Nagbbigay diin ito sa mensaheng dulot ng akda. Alin ito?

A. Moralistiko C. Naturalismo

B. Makabayan D. Historikal

28. Alin ang pinakamalapit na salin ng “You are the apple of my eye.”

A. Ikaw ay maganda sa paningin. C. Ikaw ang paborito ko.

B. Ikaw ay mansanas sa aking mata. D. Ikaw ay isang mansanas.

29. Piliin ang literal na salin mula sa sumusunod. “They patch up and kiss.”
A. Sila ay nagkabati na C. Sila ay nagtakip at nagsaya

B. Sila ay naghalikan D. Sila ang nagtagpo at naghalikan

30. Ang alpabetong Filipino ay binubuo ng ilang letra?

A. 31 C. 28

B. 20 D. 21

SOCIAL SCIENCE FOR BEED

1. Which human rights violation is a deliberate and systematic extermination of an ethnic or


national group?

A. Parricide C. Politicide
B. Homicide D. Genocide

2. Are rights absolute?

A. It depends on the situation.

B. Yes, they are absolute.

C. No, they are limited by the rights of others.

D. It depends on the right exercised

3. In 1989, it became the first country to sanction homosexual marriage and to give
homosexuals most of the legal rights accorded to heterosexuals.

A. Netherland C. Denmark

B. Germany D. Finland

4. Which is the country, help in the founding of OPEC, being one of the world’s leading oil
producers?

A. Venezuela C. Saudi Arabia, Jordan

B. Ecuador D. Mexico

5. Which is the second most populated country in the world?

A. India C. America

B. Canada D. China

6. India leads in the production of ______________ in the world

A. tea C. spices

B. wheat D. tobacco

7. Do the “unborn” have the right to life?

A. Yes, but only those 6 months old

B. No, they are not yet born

C. Yes, they have

D. No, they are not yet human

8. “My right ends where the rights of others begin.” What does this imply?

A. Rights are inviolable C. Rights have limits

B. Rights are absolute D. Rights are alienable

9. Which of UNESCO’S pillars of education is most related to peace education?

A. Learning to live together C. Learning to know

B. Learning to be D. Learning to do

10. What does EDSA 1 in our history as people prove? It was shows in ______________.

A. preference for the poor C. “people power”


B. armed violence D. information

11. Which does long lasting peace involve?

I. Social integration

II. Economic development

III. Social exclusion

IV. Political participation

A. I, II and IV C. II only
B. I and II D. I, II and III

12. A violation of human rights in the Philippines which was demonstrated by activists recently
and has gained notice of the international community is the _________________.

A. cheating in the most recent election

B. mysterious killings of journalists

C. exerbistant increase in the prices of commodities

D. imposition of oppression taxes

13. Which peace efforts are most governments engaged in when they have representatives in
other countries to promote international cooperation and harmony?

A. Disarmament C. International organization

B. Diplomacy D. Collective security

14. Which values are rooted in the Filipinos awareness of his/ her social-relatedness??

I. Delicadeza

II. Pakikipagkapwa tao

III. Pakikiramay

A. II only C. I, II and III


B. III only D. II and III

15. Which type of cooperative promotes thrift among members and generate funds to lend to its
members?

A. Credit cooperative C. Consumers cooperative

B. Service cooperative D. Producers cooperative

16. How does the environment tax peoples labor?

A. Through value added tax

B. Through inheritance tax

C. Through income tax

D. Through real estate tax

17. Which is the primary factor that drives most Filipinos to leave their families and work
abroad?
A. illiteracy C. Insurgency

B. Poverty D. Dissatisfaction

18. Our opposition to divorce could be an influence of the _________________.

A. protestant religion introduced by the Spaniards

B. educational system introduced by the Americans

C. Roman catholic religion introduced by the Spaniards

19. Because of the onset and proliferation of AIDS cases, which social problem gained renewed
attention and concern?

A. Child Labor C. Pornography

B. Prostitution D. Drug Addiction

20. The words “dangal”, “delicadeza” and “palabra de honor” prove that the Filipino is
___________________.

A. respectful C. reflective

B. ready to sacrifice D. aware of his worth

GEOGRAPHY for BEEd

1. Which is the largest of all continents on earth’s land surface?

A. Europe C. North America


B. Asia D. Africa

2. Where are the world’s major religions such as Hinduism, Buddhism and Judaism being
practiced?
A. Asia C. South America

B. Africa D. Europe

3. The highest peak in Asia is ____________________.

A. Mont Blanc C. Mt. Everest

B. Rocky Mountains D. Mt. Kilimanjaro

4. The region referred to as “Far East” are ___________________.

A. India, Pakistan C. Saudi Arabia

B. China, Korea, Japan D. Russia, Kazakhstan

5. The Prime Meridian passes through ___________________.

A. Rome C. London

B. Warsaw D. New York

6. Chernobyl experienced a nuclear explosion in 1986. It is located in __________________.

A. Russia C. Lithuania

B. Ukraine D. Latvia

7. The two big bodies of water that surround the Philippines are ___________________.

A. Pacific and Indian Oceans C. Pacific Ocean and South China Sea

B. Aractic and Indian Oceans D. Atlantic Ocean and South China

8. Our country is considered an archipelago which is composed of ___________________


islands.

A. 1, 105 C. 7, 110

B. 1, 106 D. 7, 107

9. The exact location of our country is _______________________.

A. 4º-25ºN and 110º-125ºE C. 5º-25ºN and 111-126ºE

B. 4º-23ºN and 116º-127ºE D. 5-23ºN and 113-127ºE

10. The famous river systems Rhine, Danube, Thames are located in __________________.

A. Europe C. Africa

B. Souhteast Asia D. South Amarica

11. Our country’s highest peak is _______________________.

A. Mt. Pulog C. Mt. Kanlaon

B. Mt. Bulusan D. Mt. Apo

12. Which are our official summer season and wet season?

A. March to June; July to October

B. March to May; June to September


C. April to June; July to November

D. April to May; June to September

13. As of 2009, how many cities do we have?

A. 125 C. 145

B. 135 D. 155

14. In which continent are the largest metropolitan areas like Mexico City, New York City and
Chicago located?

A. South America C. North America

B. Europe D. Asia

15. An effective teaching employed by Social Studies teachers is one wherein the students
imagine a country or place they like to visit one day. They are supposed to write about the
scenic landscape, cultural institutions and hospitable people living there. This strategy may be
classified as ________________.

A. conceptualizing C. visualizing approach

B. “guided imagery” activity D. inferential approach

16. The subject Geography deal with “written visuals” in learning about specific locations and
their areas, capital cities and famous landforms therein. The best learning activity could be
___________________.

A. sketching as in locations

B. tracing using lines

C. constructing figures

D. illustrating with cut-outs

17. Interesting natural sites as the country’s famous Pagsanghan Falls, Bohol Chocolate Hills
and Taal Lake can best be taught through ____________________.

A. clippings C. field visits

B. magazine D. textbooks

18. Name the leats developed and poorest country in Central America, also known as “Banana
Republic.”

A. El Salcavador C. Guatemala

B. Nicaragua D. Honduras

19. A Social Science teacher wants her students to examine unique events, a pressing problem
and solutions, culture, what strategy will be effective?

A. Case Studies C. Field Visits

B. KWL D. Cooperative learning

20. If the task is to decide which of two competing documents of an event, person or institution
about the past has the correct interpretations, what strategy do they use?
A. Fishbowl C. Dueling Documents

B. Case Studies D. KWL

MATHEMATICS SCIENCE SOCIAL SCIENCE

1. B 1. C 54. A 1. A 51. C
2. A 2. B 55. C 2. C 52. C
3. A 3. C 56. A 3. B 53. B
4. B 4. C 57. A 4. D 54. A
5. D 5. D 58. D 5. A 55. B
6. B 6. B 59. B 6. D 56. A
7. A 8. C 60. B 7. B
8. C 9. B 61. C 8. B ENGLISH
9. D 10. C 62. D 9. C 1. D 43.C

10. A 11. D 63. C 10. C 2. A 44. B


11. A 12. A 64. C 11. A 3. B 45. C
12. B 13. D 65. A 12. D 4. B 46. A
13. C 14. B 13. B 5. D 47. B
14. D 15. B 14. D 6. C 48. D
15. D 16. C 15. C 7. D 49. B
16. D 18. B 16. C 8. A 50. B
17. C 19. C 17. B 9. B 51. D
18. D 20. A 18. A 10. C 52. B
19. D 21. B 19. D 11. D 53. B
20. A 22. A 20. C 12. B 54. A
21. C 23. D 21. B 13. B 55. C
22. C 24. D 22. D 14. C 56. A
23. C 25. D 23. D 15. D 57. B
24. D 26. C 24. D 16. D 58. A
25. C 28. B 25. B 17. B 59. A
26. C 29. C 26. C 18. B 60. D
27. C 30. A 27. B 19. C 61. B
28. D 31. D 28. A 20. C 62. B
29. B 32. B 29. D 21. B 63. C
30. C 33. C 30. D 22. B 64. D
31. A 34. A 31. B 23. D 65. A
32. A 35. C 32. B 24. B 66. A
33. A 36. B 33. A 25. A 67. D
34. B 37. B 34. A 26. C 68. B
35. A 38. D 35. C 27. B 69. B
36. B 39. B 36. D 28. A 70. A
37. B 40. C 37. B 29. A 71. C
38. A 41. D 38. D 30. D 72. B
39. B 42. A 39. D 31. B 73. D
40. C 43. A 40. A 32. A 74. A
41. B 44. B 41. B 33. B
42. C 45. B 42. B 34. D
43. B 46. C 43. C 35. D
44. C 47. D 44. C 36. B
45. A 48. D 45. A 37. A
46. A 49. C 46. B 38. B
47. A 50. C 47. C 39. D
48. C 51. A 48. D 40. C
49. A 52. A 49. A 41. B
50. C 53. B 50. B 42. D

PILIPINO PROFESSIONAL EDUCATION

1. D 53. A 1. B 53. B 106. C 158. D


2. B 54. B 2. D 54. C 107. A 159. D
3. C 55. C 3. C 55. A 108. D 160. A
4. D 56. A 4. D 56. B 109. B 161. B
5. A 57. A 5. A 57. D 110. A 162. A
6. A 58. D 6. A 58. C 111. C 163. D
7. B 59. D 7. D 59. A 112. A 164. A
8. A 60. C 8. A 60. A 113. D 165. B
9. A 61. B 9. C 61. A 114. B 166. A
10. B 62. B 10. D 62. B 115. B 167. B
11. B 63. C 11. B 63. C 116. A 168. A
12. C 64. B 12. D 64. A 117. D 169. C
13. B 65. A 13. C 65. C 118. A 170. B
14. B 14. A 66. A 119. A 171. D
15. D 15. B 67. B 120. B 172. D
16. A 16. D 68. C 121. A 173. B
17. D 17. A 69. A 122. A 174. A
18. C 18. D 70. D 123. C 175. A
19. B 19. B 71. B 124. A 176. A
20. D 20. D 72. C 125. B 177. B
21. B 21. D 73. B 126. A 178. A
22. C 22. B 74. D 127. B 179. C
23. B 23. C 75. A 128. C 180. D
24. A 24. B 76. B 129. A 181. A
25. C 25. D 77. A 130. A 182. C
26. D 26. A 78. A 131. B 183. A
27. B 27. B 79. C 132. D 184. A
28. C 28. A 80. A 133. A 185. A
29. A 29. C 81. B 134. A 186. A
30. A 30. D 82. C 135. C 187. A
31. B 31. B 83. A 136. A 188. D
32. B 32. A 84. B 137. A 189. A
33. D 33. B 85. D 138. A 190. A
34. B 34. D 86. A 139. B 191. B
35. D 35. D 87. D 140. C 192. A
36. C 36. C 88. B 141. A 193. B
37. B 37. D 89. A 142. B 194. B
38. D 38. C 90. A 143. B 195. A
39. A 39. A 91. C 144. A 196. A
40. D 40. C 92. A 145. A 197. A
41. A 41. D 93. B 146. B 198. A
42. B 42. B 94. D 147. A 199. A
43. D 43. C 95. D 148. B 200. A
44. C 44. A 96. B 149. D 201. D
45. A 45. A 97. A 150. C 202. D
46. C 46. B 98. C 151. A 203. A
47. B 47. C 100. C 152. A 204. A
48. A 48. A 101. A 153. D 205. A
49. C 49. B 102. A 154. A 206. A
50. D 50. D 103. B 155. D 207. A
51. A 51. B 104. A 156. D 208. B
52. C 52. A 105. A 157. B 209. D

210. A 264. B CONTENT SPETILIZATION A.BIOLOGY AND ENVIRONMENTAL SCIENCE


211. A 266. D 1. A
212. D 267. D 2. B
213. A 268. B 3. C
214. A 269. B 4. D
215. B 270. A 5. A
216. A 271. A 6. B
217. D 272. B 7. A
218. A 273. A 8. C
219. B 274. A 9. B
220. D 275. A 10. C
221. A 276. A 11. B
222. B 257. B 12. D
223. D 258. C 13. A
224. B 269. A 14. A
225. A 270. A 15. C
226. A 271. A 16. B
227. C 272. B 17. A
228. A 278. B 18. A
229. A 279. D 19. C
230. B 280. A 20. B
231. C 281. A 21. C
232. A 282. A 22. D
233. A 283. B 23. A
234. A 284. A 24. A
235. A 285. A 25. C
236. A 286. A 26. B
237. D 287. A 27. A
238. B 288. A 28. B
239. D 289. D 29. D
240. A 290. A 30. C
241. A ` 291. D 31. C
242. A 292. A 32. A
243. A 293. A 33. B
244. B 294. B 34. C
245. A 295. A 35. A
246. A 296. A 36. B
247. B 297. A 36. C
248. A 298. A 37. A
249. A 299. A 38. A
250. C 300. B 39. A
251. B 301. B 40. C
252. B 302. A
253. B 303. A
254. A 304. A
255. B 304. A
256. A 335. A
257. B 36. A
258. C
259. A
260. A
261. A
262. B
263. A

B. Physical, Sciences, Earth

Science for BEEd ENGLISH for BEEd FILIPINO for BEEd

1. B 1. A 1. B
2. C 2. A 2. A
3. B 3. B 3. C
4. A 4. C 4. A
5. C 5. B 5. C
6. B 6. C 6. C
7. D 7. B 7. B
8. B 8. C 8. C
9. D 9. B 9. C
10. C 10. C 10. A
11. A 11. A 11. A
12. C 12. A 12. B
13. A 13. A 13. C
14. B 14. C 14. C
15. A 15. D 15. A
16. B 16. A 16. D
17. D 17. C 17. A
18. C 18. A 18. B
19. D 19. D 19. C
20. B 20. B 20. B
21. C 21. B 21. C
22. B 22. A 22. A
23. A 23. C 23. C
24. C 24. D 24. B
25. D 25. A 25. C
26. B 26. B
27. C 27. A
28. B 28. C
29. A 29. D
30. C
30. B SOCIAL SCIENCE for BEEd
31. C 1. A
32. D 2. B
33. C 3. C
34. C 4. A
35. A 5. D
36. B 6. A
37. A 7. C
38. A 8. B
39. B 9. A
40. C 10.C
41. A 11. A
42. C 12. B
43. B 13. A
44. B 14. C
45. C 15. A
46. B 16. C
47. D 17. B
48. C 18. C
49. A 19. B
50. B 20. D

GEOGRAPHY for BEEd

1. B
2. A
3. C
4. B
5. C
6. B
7. C
8. D
9. B
10. A
11. D
12. A
13. B
14. C
15. B
16. A
17. C
18. D
19. A
20. C

Das könnte Ihnen auch gefallen